Test Bank for PMHNP Boards 2023 Questions and Answers

What is true of interprofessionality?

All health professionals have individual professional competencies

What is the significance of an experimental p-value of 0.05

There is a 5% chance of a similar result occurring by chance alone

What percentage of children with oppositional defiance disorder will develop conduct disorder?

30%

In the DSM-5, selective mutism is now considered what type of disorder?

anxiety disorder

Symptoms of selective mutism must persist for at least

1 month

Common competencies

overlap more than one health care professional, although not necessarily all health care professionals, and are competencies expected of all health care professionals

Complementary competencies

enhance the qualities of other professions in provision of care

Collaborative competencies

those that each individual profession must possess to work with others, including those who practice within different specialties within a profession.

Interpersonal collaboration is _______ centered

patient

Cognitive therapy is recommended for patients that have

negative cognitive distortions
irrational beliefs
faulty conceptions

Humanistic Therapy focuses on assisting patients with

self-actualization and self-directed growth

Behavioral Therapy focuses on

changing maladaptive behaviors through the use of behavioral modification techniques

Existential therapy is for

reflection on life
confronting self

120 who got diarrhea ate lettuce. 80 with diarrhea did not eat lettuce. 40 without diarrhea ate lettuce. 160 without diarrhea ate lettuce. What is the odds ratio of eating lettuce and developing diarrhea?

6 ((120X160)/(80×40))

The New Freedom Commission on Mental Health (NFCMH) released a report that suggested

implementation a national campaign to reduce the stigma of seeking care
Focusing on promoting recovery and building resilience

The efficacy of mental health treatments was found to be well documented in

influential report issued by US Surgeon General in 1999

State of high negative emotion and pessimism

neuroticism

Commonly known as an optimistic outlook on life They are are outgoing, like social interaction, respond to external rewards, have more active dopamine networks, and are responsive to positive reinforcement.

Extraversion (high levels protect from psychiatric illness)

Introverts are more likely to be _______ motivated and more likely to respond to _______ reinforcement

internally
negative (more likely to interact with internal motivation/self-image)

Interpersonal trait of cooperation, easy going nature. Tend to have smooth relationships often at the expense of self-assertion

Agreeableness

Associated with self-control and a focused, organized approach to life. Are achievement-oriented goal setters who can delay immediate gratification to obtain their long term desired outcome. They are considered responsible, reliable, and dependable.

conscientiousness

In behavioral couples therapy, the initial sessions are spent doing a functional analysis of behavior (FAB). What is true about FAB?

FAB is concerned only with observable stimuli connected to operant behavior
FAB is based on operant conditioning (identifying antecedent stimuli, operant behaviors, and consequent stimuli)
The goal of FAB is modifying willful operant behavior

Gestalt theory deals with _____, not _____

perceptual psychology
behavior

Medication that can delay ejaculation in patients with premature ejaculation

Paroxetine (and other SSRIs)
(others include:
Topical anesthetic
Tramadol
Phosphodiesterase-5 inhibitors)

First line treatment for lithium induced diabetes insipidus

Amiloride (K potassium diuretic) in addition to dc lithium when possible.

Ways to reduce polyuria associated with lithium toxicity

reduce lithium dose to minimum effective dose
take once daily at night (may be 5-30% lower than when given in divided doses over the day)
Encourage adequate fluid intake

What is true about ECT in Parkinson’s disease

it’s more likely to cause transient delirium (worsened dyskinesias induced by l-dopa)

Lithium effects with Parkison’s disease

may worsen tremor

nonselective MAOIs decrease the efficacy of ______ and increase the risk of a __________

l-dopa products
hypertensive crisis

Selegiline is selective for

MAO-B
(after 10mg there is an increase in risk of serotonin syndrome if given with other meds that increase serotonin)

SSRI’s can _______ Parkinsonism tremor

worsen (and decrease ‘off time’)

Process describes the mechanism by which the hippocampus stores memories

long-term potentiation

loss of memory relating to a particular category of information (inability to recall important autobiographical information, usually of a traumatic or stressful nature, that is inconsistent with ordinary forgetting)

systematized amnesia

Lack of memories from a circumscribed period of time, usually immediately following a traumatic event

localized amnesia

Person can recall some but not all of events from a particular time period

selective amnesia

Lack of memory of the patient’s entire life

generalized amnesia

The drugs that are FDA approved for GAD

Paroxetine
Citalopram
Escitalopram

Venlafaxine
Duloxetine

Buspar

A surrogate has the right to terminate pregnancy if

the pregnancy threatens her health or life
or
she is a genetic parent

If the surrogate receives both ovum and sperm from couple, she loses the right to

determine whether or not child will be aborted (this is up to the couple)

Which medication is most effective for treating social phobia/social anxiety disorder, especially when refractory?

Phenelzine

A 55-year-old female patient has Stg II HTN. On exam, she has trace pedal edema bilaterally, but her lungs are clear. Her ECG reveals 2nd degree heart block. Which of the following BP medications should be avoided?

Calcium Channel Blockers

3 Contraindications to use a calcium channel blocker

1st or second degree heart block
Bradycardia
CHF

4 Side effects of calcium channel blockers

Headache
Lower extremity edema
Bradycardia
Heart Block

An NP is evaluating a 78 year old female with depression. She suspects the patient may have Parkinson’s disease. Which of the following is consistent with that diagnosis?

Postural instability
Pill-rolling tremor
Shuffling Gait
Difficulty initiating movement
Mask-like facies (difficulty initiating movement)

Writhing movements occur with

Huntington’s chorea

10 Non-motor manifestations of Parkinsons

Autonomic dysfunction
Gastrointestical dysfunction
Rhinorrhea
Sleep disturbance
Fatigue
Mood disorder
Cognitive dysfunction
Dementia
Psychosis
Hallucinations

What type of therapy is most useful in depressed HIV positive outpatients?

Interpersonal psychotherapy

What type of therapy is most useful in patients with borderline personality disorder?

Dialectical behavioral therapy

The NP is caring for a patient dying of lymphoma. The pt’s wife states that the pt has been mismanaged and that another practitioner must be assigned to the patient for him to improve. This NP suspects that this individual is in which stage of grief according to Elizabeth Kugler-Ross?

Bargaining

Involves resisting or refusing to remember information that has been supplied

Denial

Pica and spoon shaped nails are associated with

Iron-deficiency anemia (microcytic)

2 common causes of microcytic anemia

iron deficiency
anemia of inflammation/chronic disease (TB, HIV, endocarditis, RA, Chrohn’s DM, kidney disease, CA)

(Other causes: lead toxicity, alcohol/drug use, zinc excess (copper deficiency))

Initial symptoms of B12 deficiency

Generalized weakness
Paresthesias
Possible ataxia

Late symptoms of B12 deficiency

Memory impairment
Depressed mood

5 P’s of Vitamin B12 deficiency

Pancytopenia

(Peripheral and Posterior spinal cord) neuropathy

Pyramidal tract signs

Papillary atrophy of tongue (atrophic glossitis)

2 causes of megaloblastic macrocytic anemias

Vitamin B12 deficiency
Folate deficiency

What neurotransmitter is thought to play the most significant role in social phobia?

dopamine

What neurotransmitter is thought to mediate social interest, gregariousness, confidence and sensitivity to rejection?

dopamine

According to Gonzalez-Jaimes and Turnbull-Plaza (2003, which type of therapy with psychocorporal, cognitive, and neurolinguistic components and a holistic focus is significantly more effective in patients diagnosed with adjustment disorder?

mirror psychotherapy

The ________ is part of the treatment to encourage a patient to accept his physical condition

mirror

Discussing medical diagnoses and the reaction to it, as well as treatment modalities

medical conversation

Therapy that emphasizes personal responsibility, focusing on the individual’s experience at the present moment, the therapist-patient relationship, the context of the person’s life, and adjustments people make as a result of their overall situation

Gestalt Therapy

PANDAs is associated with _________ infections and results in _______ symptoms

Group A beta-hemolytic strep
OCD

After camping trip, pt began to have a petechial rash on her legs/trunk, parts of which are turning dark red and purple, temp 102.8, HR 1115, BP 88/50, mm pain, neck stiffness, severe HA. What is the dx?

meningococcemia

1 mg of Xanax = _____ mg of Ativan

2

1mg of Xanax = ____mg of phenobarbital

30

1mg Xanax = ____ mg of Clonazepam (Klonopin)

1

1mg Xanax = ____ mg of chlordiazepoxide (Librium)

25

1mg of Xanax = ____ mg of diazepam (Valium)

10

Pt wanting something immediate and non-addictive for breakthrough anxiety

Hydroxyzine

How long do GAD symptoms need to occur to make the diagnosis?

6 months

First lab test to check for Sjogren’s

Anti-nuclear antibody tests

To diagnose Cyclothymic disorder, it must be present for _______ in adults

2 years

To diagnose Cyclothymic disorder, it must be present for _______ in pediatrics

1 year

4 physiological disturbances that may explain some of the symptoms of panic

impaired lactate metabolism
abnormality of locus coeruleus (Ser, NE, GABA, Dop, cholecystokinin)
carbon dioxide hypersensitivity (hyperventilation)

A 45-year-old man presents to the ER for “panic attack”. He appears agitated, confused, and dizzy. He became more agitated and combative with staff upon further questioning. He states he was prescribed Xanax 0.5mg 3x daily by outpatient psychiatrist. His vitals are WNL. What is most likely his dx

Benzodiazepine intoxication

Anxiety, tremors, nightmares, insomnia, anorexia, nausea, vomiting, hypotension, seizures, delirium and hyperprexia are sx of. ______ which can result in _______

Benzodiazepine withdrawal
(serious medical complications including death)

Confusion, AH, VH following a period of insomnia can be a sx of

withdrawal delirium

Female patients with PTSD commonly have this comorbid diagnoses

Major depressive disorder (50%)

Percentage of people with PTSD who have alcohol use disorder

male 50%
female 27%

Percentage of people with PTSD who have simple or social phobias

30% both male and female

Percentage of people with PTSD who have persistent depressive disorder ( previously dysthymia)

20% for both male and female

Treatment plan for OCD

SSRI or Clomipramine
Psychotherapy

Normal Rinne test is for this nerve

CN VIII

What ethnic groups are at higher risk for social phobia

Native American

What ethnic groups are at lower risk for social phobia

Asian
Black
Latino

Onset of social phobia is

Usually begins in late adolescence or early adulthood (13-20 years)

3 CBT techniques for social phobias

exposure
cognitive restructuring
social skills training

Medication used to help with social phobia for public speaking

propranolol

FDA approved SSRI for social phobia

paroxetine

What type of fracture is a red flag for child abuse?

Spiral of long bone (caused by twisting of extremity)

Bruises on the torso in different stages of healing often indicate

abuse

Tanner Stage: Breast buds undeveloped with papilla elevation only. Villus hair only

I

Tanner Stage: Breast buds palpable, straight pubic hair mostly on labia

II

Tanner Stage: Areola begins to enlarge and pubic hair becomes darker, coarser, and curly

III

Tanner Stage: Areola forms a secondary mound on the breast and hair is of adult quality

IV

Tenderness at McBurney’s point is suggestive of

appendicitis

About 30% of patients with this disorder have OCD thoughts

Tourette’s disorder

Rx that can cause anxiety

Steroids (prednisone)
Albuterol
Amphetamines
Appetite suppressants

Medical conditions that can cause anxiety

Adrenal tumors
Alcohol use disorder
Hypoglycemia
Hyperthyroidism
Cushing’s disease
Arrhythmias
Parathyroid Tumors
Some types of seizures

Depression screening tool used specifically for pregnancy and postpartum

Edinburg Depression Scale

How long do symptoms have to be present to diagnose PTSD

1 month

Lab value that would indicate that parathyroid was removed during thyroidectomy

Low Calcium

For PCOS, which medication will help with hirsutism?

Spironolactone

Lab that is an indicator of inflammation

C-reactive protein

Pain medication that inhibitis serotonin and NE reuptake (risk of serotonin syndrome if also on antidepressants)

Tramadol

an over-the-counter nonnarcotic drug found in cough preparations which inhibits serotonin reuptake and could cause serotonin syndrome in patients on SSRI’s

Dextromethorphan (Delsym)

Pre-cursor for Serotonin production

L-tryptophan

In panic disorder, 1 of the attacks is followed by at least ______ of persistent worry about having additional attacks, worry about implications of attacks or its consequences, or significant change in behavior related to attacks.

1 month

The origins of panic disorder include

childhood trauma

With delayed onset of PTSD, how much time must have past from trauma to onset of symptoms

6 months

First line medication for polymyalgia rheumatica

oral prednisone

Acute stress disorder is diagnosed if the characteristic symptoms present within what period of time after experiencing the traumatic event?

1 month

In what Tanner’s stage will a male’s penis grow more in length than width

III

In what Tanner’s stage will a male’s scrotum begins to grow and redden. Where pubic hair will be straight, fine and sparse

II

In what Tanner’s stage will a male’s pubic hair look more like an adult, but will not cover as much area. Where penis will thicken, testes and scrotum continue to grow.

IV

Brain structure R/T development of GAD

Amygdala

What circumstance(s) are often correlated to the onset of panic disorder

separation from family
serious illness of friend/family
accident
major relationship loss/change
abusing substances
postpartum

Lifetime prevalence of anxiety disorders in the US

25-35% (18% prevalence)

Buspar needs ______ to have similar efficacy to benzodiazepines

4 weeks

How many panic symptoms must be present for diagnoses.

4

Part of Brain responsible for smell

Limbic

Part of brain (lobe) responsible for visual functions

Occipital

Part of brain (lobe) responsible for sensory integration and somatosensory function

Parietal

Part of brain (lobe) responsible for executive function, social conduct, judgement, and some motor function

Frontal

Occasional crossing of eyes is not normal after the infant is older than ___________

2 months

If strabismus is left untreated, what may result?

ambylopia (affected eye decrease in vision)

What would be the normal testosterone level of a Tanner V male

265-800 ng/dl

Bactrim is contraindicated in

Glucose-6-phosphatase deficiency
Folate deficiency anemia
patients with significant hepatic impairment
pregnant patiens

Pt has severe headache behind right eye with eye pain and tearing, sees halos around light fixtures. C/o N/V. R pupil fixed more oval than round, mid-dilated and cloudy. What might be happening

Acute angle closure glaucoma (Emergency)

What STD is not reportable in all 50 states

HPV

Romberg test is a test of

balance

A child will recognize faces and smile appropriately at familiar faces by age ____

3

A typical _____ can count up to 20 with minimal assistance of an older child or parent

4 year old

A typical _____ can run/walk up and down the stairs

21 month old

A typical _____ is beginning to acquire language skills

14 month old

What herbal supplement is effective for a pt with an adjustment disorder with anxiety according to RCT

Kava Kava

What herbal supplement is effective for somatoform disorders according to RCT

St. John’s Wort

Recurrent spontaneous episodic vertigo, hearing loss. Must have aural fullness or tinnitis on one size to make dx.

Meniere’s Disease

ADPIE stands for

Assessment
Diagnosis
Planning
Implementation
Evaluation

If diagnosed with an adjustment disorder, which dx is most guarded: that of an adult or that of an adolescent?

adolescent (more likely to develop major psychiatric illness)

Gastrointestinal lithium toxicity symptoms can overlap with

normal side effects (N/V)

What is FDA approved for suicidality in schizophrenia

Clozapine

Clozapine can cause this CBC abnormality

leukopenia (agranulocytosis)

What is the dependent variable of a study examining the effects of a certain medication on hypertension?

Blood pressure

What is the independent variable of a study examining the effects of a certain medication on hypertension?

Medication

Effect of a variable that is not controlled

Confounding effect

Type of error that results in the way the experiment is conducted, often due to the expectations of the researcher

Bias

Pharmacological treatment of alcohol use disorder

Start Naltrexone at 25mg PO daily (1-hour observation for rxn). Naltrexone maintenance dose is usually 50mg PO q day or 380mg IM q 4 weeks

Naltrexone side effects

Elevated LFTs
Mild dysthymia
GI upset

Resembles GABA and is possibly an NMDA antagonist

Acamprosate

Aldehyde dehydrogenase inhibitor

Disulfiram (Antabuse)

Modest efficacy, may work better in combination with Naltrexone for alcohol use disorder

Acamprosate

No convincing efficacy found for use of this Alcohol Use Disorder pharmacological treatment

Disulfiram (Antabuse)

Most commonly reported side effect of ECT

Retrograde amnesia

Neurodevelopmental disorder of biological origin that manifests in learning difficulties and problems in acquiring academic skills markedly below age level and manifested in the early years for at least 6 months that is not attributed to intellectual disabilities, development disorders or neurological or motor disorders

Specific Learning Disorder

Specific Learning Disorders can be specified as impairment in

Reading
Written Expression
Mathematics

The MOST important component when developing evidence based guidelines

An evidence review

How does botox improve depression?

Reducing frown lines when injected into glabellar region

According to a study in the 1990s mothers of anorexic pts have an increased prevalence of this disorder

Obsessive-compulsive disorder

The AST:ALT ratio in patients with Alcoholic liver disease

<0.8
>1.5

Liver biopsy reveals fibrosis, necrosis of liver cells, and nodule formation

Liver cirrhosis

Acute condition with nystagmus, ataxia, confusion, and opthalmoplegia

Wernicke-Korsacoff syndrome

Elevated bilirubin level is often found in

Alcoholic hepatitis

Which drug’s mechanism of action includes blockage of the voltage dependent sodium channel blockers, augmentation of gamma amino butyrate activity of GABA-A receptors, antagonism of AMPA/kainate subtype of glutamate receptor, and inhibition of carbonic anydrase enzyme?

Topiramate

Ex of drugs that are partial agonists at postsynaptic serotonin receptors

Hallucinogens like LSD and MDMA

Drug associated with inhibition of catecholamine reuptake in neurons and blockage of NMDA glutamate receptors

PCP

Individuals with panic disorder are most commonly affected by which other psychiatric disorder

Agoraphobia

When is the use of transdermal selegiline not beneficial?

For patients with pheochromocytoma (excess NE can cause HTN crisis) and with planned surgery (need to dc patch for 10 days prior to elective surgery)

Responsible for the catabolism of serotonin, norepinephrine and dopamine

Monoamine oxidase (MAO)

Medication that is an irreversible MAO inhibitor that works by selectively inhibiting both MAO-A and MAO-B in the CNS with preservation of MAO-A in the gut. Unless high dosed, little need to follow tyramine restricted diet.

Selegiline

Adverse effect of carbamazepine that can be an emergency

hyponatremia
toxic epidermal necrolysis
Erythema multiform
Steven’s Johnson
Thrombocytopenia
Aplastic Anemia
Agranulocytosis
SLE
arrhythmias
Hepatitis

An NP advises a patient with a significant family hx of breast cancer to perform monthly self-examinations. What term helps to identify this measure?

Secondary Prevention

Prevention type designed for early detection of a disease so that morbidity and mortality may be reduced

Secondary Prevention

Preventing the disease or condition from occurring

Primary Prevention

WIC and promotion of workplace safety laws designated by OSHA are this type of prevention

Primary Prevention

Rehabilitation and prevention of complications of disease

Tertiary prevention

A support group for a disease or condition is this type of prevention

Tertiary Prevention

Exercise for an obese patient is this type of prevention

Tertiary

Antisocial personality disorder criteria must show a pattern of disregard of others since age ______

15

How many criteria does a pt who meets criteria for an Antisocial personality disorder have to have

3 of 7

Name the 7 criteria that pts with Antisocial personality disorder have (must meet 3 of them)

Breaking the law
Deceitfulness for personal profit or pleasure
Impulsivity/failure to plan ahead
Aggressiveness/Irritability (physical fights/assaults)
Reckless disregard for safety of self or others
Consistent irresponsibility
Lack of remorse

How old does an individual need to be in order to be diagnosed with antisocial personality disorder?

18 years

The occurrence of antisocial behavior is not exclusively during the course of

schizophrenia
bipolar disorder

What is a precursor for antisocial personality disorder

conduct disorder

List of Cluster B personality disorders

antisocial
borderline
histrionic
narcissistic

List the cluster A personality disorders

paranoid
schizoid
schizotypal

List of Cluster C personality disorders

avoidant
dependent
obsessive-compulsive

pervasive pattern of excessive emotionality and attention seeking beginning in early adulthood and present in a variety of contexts

Histrionic Personality Disorder

A pattern of angry/irritable mood, argumentative/defiant behavior, or vindictiveness must last at least _______ for Oppositional Defiant Disorder diagnosis

6 months

Oppositional defiant disorder does not include:

aggression toward people or animals
destruction of property
pattern of theft or deceit

Recurrent behavioral outburst representing a failure to control aggressive impulses.

Intermittent Explosive Disorder

In Intermittent Explosive disorder, physical aggression does not result in

damage or destruction to property
physical injury to animals or other individuals

Kubler-Ross’s stages of grief

denial
anger
bargaining
depression/sadness
acceptance

Symptoms can be associated with another medical condition

Somatic symptom disorder

Somatic symptom disorder symptoms persist for

>6 months

Despite frequent doctors visits and tests, this pt is rarely reassured and can feel that his/her medical care is insufficient

Somatic symptom disorder

In order to meet DSM V criteria for somatic symptom disorder, in addition to somatic symptoms, pts must have

maladaptive thoughts, feelings, and behaviors that define the disorder, in addition to their somatic symptoms.

The Mental Health Party Act (1996) provides

equal lifetime and annual limits for mental health care as compared to physical health care.

The zoster vaccination is recommended for _____

adults over the age of 60

The pneumococcal vaccination is recommended for _______

adults over the age of 65

The Tdap vaccination is recommended _________

every 10 years

Understanding the objective reality of a situation coupled with the motivation and emotional impetus to master the situation or change behavior is:

true insight

Form of thinking that is logical, organized, reality oriented, and influenced by the demands of the environment

second process thinking

What percentage of the population meets the criteria for 1 or more personality disorders?

15%

Study on daily exercise improving sleep quality in seniors. What is the dependent variable

Sleep quality

The federal law that allows parents to provide insurance coverage for their children up to age 26 is

Affordable Care Act (ACA)

Pedophiles attracted to female victims usually prefer females of what age?

8-10 years old

What is true of appropriation bills?

They provide for spending authority for a single fiscal year

What bills establish laws or programs and although they recommend dollar amounts in some cases, do not allocate the funds or guarantee funds for the program

Authorization bills

What bills usually originate in the House of Representatives and provide spending authority for a single fiscal year

Appropriation Bills

Provide additional funding for projects as needed thought the current fiscal years

Supplemental appropriations

Authorization bills are funded by

Appropriation Bills

What is required for entry to practice as a Nurse Practitioner

Competency in nine content domains

The NP is caring for a 16 year old who is depressed with suicidal ideation. The pt asked the nurse NOT to tell her parents. The most appropriate intervention for the nurse to do is:

Talk to parents and pt together to help the family discuss the pt’s depression

According to Mahler, which of the following stages of infant development is disrupted in children who develop borderline personality disorder

Rapproachment

According to Mahler, which of the following stages of development was disrupted int children who developed narcissistic personality disorder

Symbiotic phase

Phase during first weeks of life, characterized by total detachment and self-absorption, infant spends most of his or her time sleeping

Autistic stage

When does the symbiotic phase occur

about 5 months of age

According to Mahler, separation-individuation has 3 stages

Hatching (5-9 months)
Practicing (9-16 months)
Rapproachment (15-24 months)

Phase where infant explores the outside world but requires that the mother is present and able to support the child appropriately and emotionally in completing the task?

Rapprochment (15-24 months)

Phase in which infant develops the physical ability to separate from mother

Practicing (9-16 months)

Disturbance of the rapprochement subphrase is associated with

persistent longing for and dread of fusion with object that is thought to be secondary to aggression or withdrawal in the mother. Thought to contribute to the development of borderline personality disorder.

Phase in which infant shows increased interest in outside world

Hatching

Stage where infant recognizes his or her mother but lacks a sense of individuality

symbiotic

Narcissistic personality disorders are likely due to

inadequate soothing during the symbiotic phase and inadequate refueling during the separation-individuation

Describes the child’s understanding that his or her mother is a separate individual and he or she is also separate

Object constancy

Object constancy leads to the formation of

internalization, which will allow the child to have an internal representation of mother in turn allowing healthy separation, exploration, and development of self-esteem

The three most common causes of secondary obesity

Cushing’s syndrome
Hypothyroidism
Genetic conditions

Compassion, a desire to do good

beneficence

Avoidance of harm

Nonmaleficence

the equal and fair distribution of resources, regardless of other factors

Justice

Dedication, fairness, honesty, advocacy and commitment to patients. It involves an agreement to keep promises and commitments based on the virtue of caring.

Fidelity

Most widely used tool for assessing the healthcare safety culture within an organization

Culture of Patient Safety Assessment developed by AHRQ

The Iowa Model of EBP (evidence-based practice) is characterized by which statement?

It is important to determine if an identified issue is a priority of the organization

The trigger for change can be

Knowledge focused or Problem focused

Identification of a problem and determination of its priority

first step in Iowa Model

Review and critique of available literature

Second step in Iowa Model

Identification of research evidence to support a change in clinical practice

3rd stage of Iowa Model

Implementation of change

4th step of Iowa Model

Monitoring of Outcomes

5th and final step of Iowa Model

Knowledge-focused triggers include

new research
practice guidelines

Problem-Focused Triggers include

Clinical problems
Issues in risk management

The first step in getting a bill to the floor is?

Drafting a bill

Probability results of study are due to chance

P value

Medicare part A coverage includes

1. hospital care and medicines received in hospital
2. SNF
3. nursing home care
4. hospice
5. home health services

Sampling used when the proportion of the study population belonging to each group of interest in known, to obtain an adequate study sample from a small group in the study population.

Stratified sampling

Selection of groups under study is random

Cluster sampling

Choosing individuals at regular intervals from a sampling frame by selecting a number to determine where to begin selecting individual subjects from a list.

Systematic sampling

Making a numbered list of all units in a population from which you want to draw a sample, deciding on the sample size, and selecting the required number of sampling used using a table of random numbers or a lottery method

Simple random sampling

What is the prevalence of borderline personality disorder in psychiatry outpatients?

10%

What is the prevalence of borderline personality disorder in psychiatric inpatients

20%

Diagnosis of borderline personality disorder includes 5 or more of the following criteria

frantic avoidance of abandonment
unstable relationships
disturbance of identity
recurrent suicidality
unstable mood/affect
chronic feelings of emptiness
intense bouts of difficult-to-control anger
transient paranoia or dissociation

Pervasive pattern of social inhibition, feelings of inadequacy, and hypersensitivity to negative evaluation

Avoidant

Pervasive pattern of detachment from social relationships and a restricted range of expression

Schizoid

Pervasive distrust and suspiciousness of others such that their motives are interpreted as malevolent

Paranoid

Pervasive pattern of social and interpersonal discomfort with, and reduced capacity for, close relationships as well as cognitive or perceptual distortions and eccentricities

Schizotypical

Pervasive pattern of grandiosity, need for admiration, and lack of empathy

Narcissistic

Pervasive and excessive need to be taken care of that leads to submissive and clinging behavior and fears of separation

Dependent

The prevalence of borderline personality disorder in primary care

6%

The prevalence of borderline personality disorder in the general population

1.5-5.9%

What best determines the effectiveness of an educational intervention?

Pt’s behavior modification and compliance rates

After suffering the loss of a child, more than 50% of parent couples go through the following

Divorce

Rubbing against non-consenting people

frotteurism

Fantasies about watching unsuspecting individuals undressed or in sexual activity

voyeurism

Use of non-living objects for sexual stimulation

fettishism

Exposing one’s genitals to strangers

exhibitionism

Besides therapy in Borderline Personality Disorder, impulse behavioral dyscontrol symptoms that present serious risk to patient are treated by adding

an antipsychotic to an SSRI

In borderline personality disorder, cognitive-perceptual symptoms respond best to

low dose antipsychotics

Impulsive-behavioral dyscontrol symptoms in borderline personality disorder

aggression
self-mutilation
reckless spending
promiscuity
substance abuse

Affective dysregulation in borderline personality disorder is treated with

SSRI or venlafaxine

For histrionic personality disorder, which type of therapy helps them focus on their unconscious motivations for being unable to commit to a stable and meaningful relationship, and instead seeking out disappointing partners

psychodynamic

Temporal lobe abnormalities and differences in the corpus callous and thalamic nuclei have been identified on MRI studies of patients with which condition

schizotypical personality disorder

Schizotypical personality disorder can be differentiated from schizophrenia by the absence of

psychotis

Prevalence of schizotypical personality

3% (more cases occur in biological relatives with schizophrenia)

Affective aggression can be treated with which medication?

Lithium

Which personality disorder is often present with pts with anorexia-nervosa who restrict rather than binge-purge

Obsessive-compulsive

A type of identification that is seen often in borderline personality disorder pts

projective identification

3 steps of projective identification

1. pt assigns component of self to another individual
2. pt manipulates this individual into integrating this component into his or her consciousness
3. process leads to increased feeling of closeness between parties

Analyst’s emotional response to pt

countertransference

When pt tends to identify aspects as either completely good or completely bad about a person

Splitting

Mental process that gives rise to lack of connection in the pt’s thoughts, memory, and sense of identity

Dissociation

Substance that is found in lower concentrations in the CSF of impulsive, violent individuals

5-HIAA

In addition to an assessment of pathological personality traits, alternate model in Section III of the DSM-5 requires an assessment of which category or a personality disorder diagnosis

level of impairment in personality functioning

A diagnosis of a personality disorder who require at least a level _______ impairment

2 (“moderate”)

What is the prevalence of obsessive-compulsive personality disorder in the general population

5-10%

Which personality disorder’s criteria includes magical thinking

Schizotypical

Females with _________ have a virilized clitoris

Congenital adrenal hyperplasia (CAH)

A higher degree of ________ has been reported in individuals with CAH, which correlates to the degree of virilization

homosexuality

A 2-year old girl with 21-hydroxylase deficiency undergoes genitoplasty. Which statement is true concerning her condition

There is a correlation between virilization degree and gender dysmorphia

What is the prevalence of paranoid personality disorder among psychiatric patients?

10-30%

Gender identity usually begins to form by this age

2

Anti-hypertensive effects on sexuality

impaired vaginal lubrication and erectile dysfunction d/t antiadrenergic effects

Spironolactone effects on sexuality

decreased desire secondary to decreased testosterone

Estrogen effects on sexuality

decreased desire secondary to decreased testosterone

Steroid effects on sexuality

decreased desire secondary to decreased testosterone

TCA effects on sexuality

erectile dysfunction or impaired vaginal lubrication secondary to anticholinergic effects

MAOI effects on sexuality

erectile dysfunction or impaired vaginal lubrication secondary to anticholinergic effects

Antipsychotics effects on sexuality

erectile dysfunction or impaired vaginal lubrication secondary to anticholinergic effects

SSRI effects on sexuality

inhibition of arousal and orgasm by increased serotonergic activity

The diagnosis of a personality disorder requires a pattern of inner experience and behavior that deviates from the expectations of the individual’s culture in 2 or more of which 4 areas?

cognition
affectivity
impulse control
interpersonal functioning

Individual is only aroused by and can only achieve orgasm through a single type of sexual contact, usually relegated to a specific anatomical location

Partialism

Individuals who attempt to avoid penetrative sex due to a lack of interest may have what?

Partialism (a type of paraphilia)

Prevalence of antisocial personality disorder in females in the community

1%

2 common defense mechanisms seen in obsessive compulsive personality disorder

isolation of affect
intellectualization

Paraphilia are almost never diagnosed in

females
(and rarely in a clinical setting)

Age range that is typical for a person that performs frotteurism

15-25 (declines after that)

Pts with a history of childhood separation are more prone to this personality disorder

dependent

Pts with dependent personality disorder commonly experience these 2 things from their caretaker?

over-involvement
intrusive behavior

Childhood experience common in pts with dependent personality disorder

social humiliation – leads them to doubt in their own ability to function independently of others

Pts with obsessive compulsive personality disorder often had parents who were

emotionally withholding
overprotective
over-controlling

People with obsessive compulsive personality disorder develop their symptoms as a strategy to

avoid punishment (they were often punished by parents and rarely rewarded)

Patients with schizoid personality disorder typically from families that are

emotionally reserved
highly formal
aloof
impersonal
(likely were provided inadequate affection prompting their disinterest in forming close relationships later in life)

Patients with this personality disorder displayed excessive shyness and fear when confronted with new people and situations as children

avoidant

Many of the patients have a history of painful early experiences and chronic parental criticism

avoidant personality disorder

Symptoms of personality disorders are usually consistent with patient’s

internal sense of self, which is called ego-synodic

What is the term for how easily an individual acts on an initial, emotionally based evaluation of events and people

Constraint

Purest expression of the biological basis of personality

Temperament

What are the 2 distinct temperaments Kagan and colleagues (1988) identified

inhibited
uninhibited

Which of the following medications is the first-line treatment option for a personality change secondary to epilepsy that results in aggressive episodes?

Carbamazepine

What is the second-line treatment option for a personality change secondary to epilepsy that results in aggressive episodes

benzodiazepines

What is contraindicated in the treatment of aggressive personality changes due to epilepsy

antipsychotics

females that have this are usually infertile have low estrogen levels but do not have higher levels of homosexuality

Turner syndrome(45XO)

females with this are typically indistinguishable from other females except they are slightly taller and have increased learning difficulties and low-normal intelligence

Triple X syndrome

46XY with complete androgen insensitivity have

female gender identity equivalent of 46XX individuals

Congenital adrenal hyperplasia is associated with increased prenatal

androgen exposure

5-alpha-reductase deficiency is associated with

higher rates of homosexuality in affected females
a cause of pseudohermaphroditism

Gender identity is generally set by age ____ and very difficult to change

3

First line agents for treating impulsivity and aggression in personality disordered patients

SSRIs

Explain what the following Pyramid Tract Signs result from:
spasticity
weakness
slowing of rapid alternating movements
hyper-reflexia
Babinski sign

Vitamin B12 deficiency

The only TCA levels that can be reliably followed are for

desipramine
imipramine
nortriptyline

A transgender male is sexually attracted to men/masculinity. What is his sexual orientation?

Androphilic

A transgender male is sexually attracted to females/femininity. What is his sexual attraction?

Gynephilic

A transgender female is sexually attracted to females/femininity. What is her sexual attraction?

Gynephilic

A transgender female is sexually attracted to men/masculinity. What is her sexual attraction?

Androphilic

Attraction that is combination of androphilia and gynephilia

Ambiphilia

Asexual is synonymous with

analophilic

Part D of criteria for illness anxiety disorder

performs excessive health-related behaviors or exhibits maladaptive avoidance

Physical symptoms are less prominent, while anxiety era preoccupation with having or acquiring a serious medical condition is the core complaint

Illness anxiety disorder

In illness anxiety disorder somatic symptoms are

not present or are mild in intensity

In illness anxiety disorder, illness preoccupation has to be present for at least _____ months

6

Prevalence of illness anxiety disorder

1-5%

What are the FDA-approved medications for personality disorders?

There are none

Pts with borderline personality disorder and psychotic disorders are particularly likely to have experienced

victimization as a child and/or adult

Repeated changes in caregivers, history of neglect, avoidance of social interactions, and resistance to comforting from others is a sign of

reactive attachment disorder

indiscriminate sociability and lack of selectivity in authority figures

Disinhibited social engagement disorder

Symptoms present for _______ are more consistent with PTSD rather than Acute Stress Disorder

greater than 1 month

Estimated lifetime prevalence of PTSD is

7.8%

What ethnic group of combat veterans experience a higher rate of PTSD

Latino especially Puerto Rican

PTSD rates among Native Americans are _____ the national average

3x higher

What disorder is often comorbid with body dysmorphic disorder

social phobia

Recovering completely within _______ is characteristic of a conversion disorder

24 hours

For pediatrics with conversion disorder (psychogenic seizure), what might be found in this patient’s history

She argued with her parents earlier that day over a bad report card

Conversions disorders do not show

pain symptoms

What is the typical course of illness anxiety disorder

Onset in early adulthood
has a chronic waxing/waning course

Which medications have the strongest evidence for treatment of PTSD

Paroxetine
Sertaline
(Venlafaxine also approved by FDA)

Pts with this disorder often exhibit emotional blunting that is sometimes referred to as “la belle indifference”

conversion disorder

Pts with ________ PTSD typically show a more robust clinical response to SSRIs

non-combat

In treating pts with histrionic personality disorder, which therapy type is useful for addressing provocative and attention seeking behavior?

Group therapy (pt may be unaware of behavior and it could help them if others point them out)

What term represents a stable and realistic sense of self?

Self-system

means of interpreting social situations and understanding the relational motives and actions of others

social system

the capacity to observe the self as it relates to others

self-in-relation system

What percentage of patients with neurological problems have conversion symptoms?

1-3%

Body dysmorphic disorder is associated with what risk factor and family history

Childhood neglect
OCD

These symptoms are a sign of which drug intoxication: elation
euphoria
increased physical activity
anxiety
tachycardia
arrhythmias, hypertension, mydriasis (pupil dilation), agitation
cognitive impairment

cocaine

Most serious complications of cocaine abuse

myocardial ischemia and infarction

Signs of ____________: dysphoria, psychomotor slowing, fatigue, increased appetite, craving

cocaine withdrawal

Signs of ____________:
piloerection
lacrimation and rhinorrhea
pain in joints and muscles
dysphoria
diarrhea
nausea
vomiting
dilated pupils
insomnia
autonomic hyperactivity
yawning

Opioid withdrawal

signs of ______________:
euphoria
drowsiness
impaired coordination
dizziness
confusion
feeling of heaviness in the body
slowed or arrested breathing
constipation
flushing of feelings of warmth
pinpoint pupils

opioid intoxication

Signs of ______________
Nystagmus
Dissociative symptoms
anxiety
tremor
numbness
memory loss
analgesia
psychosis
aggression
violence
slurred speech
poor coordination
hallucination

Phencyclidine intoxication

Signs of _________:
stimulation
loss of inhibition
headache
N/V
slurred speech
poor motor coordination

Inhalant intoxication

Signs of _________:
Exhilaration
loss of inhibitions
slurred speech
staggering gait
euphoria
marked motor impairment
confusion
stupor
coma

Sedative hypnotic (alcohol and benzodiazepine) intoxication

Sign of ________:
sweating
flushed face
insomnia
hallucinations
seizure
disorientation
tremors
increased autonomic activity

Sedative hypnotic (alcohol and benzodiazepine) withdrawal

Sign of ___________:
Altered states of perception and feeling
hallucinations
nausea increased
body temperature
HR and BP
pupillary dilation
poor appetite
sweating
sleeplessness
numbness
dizziness
rapid shifts in emotion
depression
anxiety

Hallucinogen intoxication

A 30-year-old female presents to the ED with severe anxiety, palpitations, restlessness, and irritability. She reports no previous history of these symptoms. Her BP is 160/100, pulse is 115 beats/min, temperature 98 degree F, and pupils are dilated What condition explains her symptoms

cocaine intoxication

Which statement differentiates the actions of cocaine and amphetamines

Cocaine prevents dopamine reuptake but amphetamines both slow dopamine reuptake and induces dopamine release

In the COMBINE trial for alcohol dependent patients acamprosate faired ____ compared to various combinations of naltrexone and combined behavioral intervention

poorly

For Alcohol dependent patients, the COMBINE trial found that

any combination of therapies including CBT, naltrexone, or both performed better than any combinations that did not include these interventions

Buprenorphine is a ______ at mu receptors and binds with ______

partial agonist
high affinity (20x stronger per gram weight of morphine)

This is resistant to cases of overdose to reversal by naloxone

buprenorphine d/t high binding affinity

DSM-5 requires ____ of 9 criteria to be met for persistent and recurrent maladaptive gambling behavior

4

What drug should be administered to address patient’s aggression on PCP

Benzodiazepines

What medications should be avoided d/t risk of hyperthermia, lower the seizure threshold, cause dystonia, or induce dysrhythmias when pt is intoxicated with PCP

haldol
chlorpromazine

Which of the following is not a feature of kleptomania

individual feels great pleasure and excitement leading to the theft (usually individual feels tension)

Chronic alcohol use disorder reflects an _______ personality

oral-dependent

Excessive alcohol use is associated with ________ traits such as

oral
dependence and depression

Men are more than _____ as likely as women to develop chronic alcohol use disorder

2x

Delirium tremens usually develops _____ from the person’s last drink, with an average durage of _____

2-4 days
<1 week

This type of head bleed presents with headaches and confusion

subdural

Most common approach to managing urges to shoplift

commit a self-imposed ban on shopping

A 16 year old attended a rave with her friends. She presents with HTN, hyperthermia, and clinical examination indicates brisk reflexes. What conditions is her presentation consistent with

Serotonin Syndrome (likely from MDMA (ecstasy))

What is the rationale behind avoiding the use of restraints in someone intoxicated on PCP

Incidence of rhabdomyolysis in patients with PCP intoxication and physical restraints is much higher than baseline but can occur with PCP alone.

With heavy cocaine use, patients can experience

a shower of lights in their central vision, as well as visual hallucinations of black dots on their skin and in the environment (coke bugs)

What is a symptom of amphetamine withdrawal

fatigue
vivid bad dreams
insomnia/hypersomnia
increased appetite
psychomotor agitation/retardation

A 29 year old male with yawning, perspiration, runny nose, dilated pupils, muscle twitching, no abdominal tenderness. Which drug is responsible for these withdrawal symptoms

Heroin

The death rate among opioid abusers is ____ greater than the non-using population

20x

Sweating, flushing, sleep disturbances, hallucinations, seizures, and mild mental status changes are early symptoms of

alcohol withdrawal 8-10 hours after last drink

LSD is derived from which of the following

Ergot fungus

Cocaine is derived from the leaves of the plant

erythroxylon coca

A 22 yo F with cystic fibrosis (CF) presents with fatigue, dysphoria, and depression. She stopped taking her meds and is wearing a baby pacifier on her necklace. Why is the pacifier significant in this pt with possible CF exacerbation?

MDMA (ecstasy) withdrawal

baby pacifiers are used at “raves” to combat

bruxism (teeth grinding) while on MDMA

Ganja is obtained from

flowering tops THC 6-20%

Marijuana is obtained from

leaves/flowers

Bhang is obtained from

dried marijuana leaves (1-3% THC)

Hashish is obtained from

resin (THC 6-20%)

In DSM-5 how many of the 11 criteria for substance use disorder must be present to meet “severe” subclassifications?

6

In DSM-5 how many of the 11 criteria for substance use disorder must be present to meet “mild” subclassifications?

2-3

In DSM-5 how many of the 11 criteria for substance use disorder must be present to meet “moderate” subclassifications?

4-5

Sx of this type of intoxication are conjunctival infection, dry mouth and increased appetite

Cannabis intoxication

What is the preferred benzodiazepine for treating withdrawal patient with chronic alcohol use disorder and a history of liver disease

Oxazepam

MRI in Korsakoff’s syndrome shows the following 6 areas of involvement

midbrain
pons
hypothalamus
thalamus
mammillary bodies
cerebellum

What vitamin is deficient in Wernicke’s encephalopathy and Korsakoff’s syndrome

Vitamin B1

The proposed addictive component of cannabis

releases dopamine in the mesolimbic system

What type of users can appear to be bipolar with mania/hypomania symptoms

anabolic steroids

Which drug of abuse is also a schedule III prescription for the treatment of narcoleptic cataplexy?

GHB (Xyrem)

Which of the following statements regarding the treatment of cocaine dependent individuals is correct

contingency management has been widely accepted and is effective
There are no FDA approved medications, no meds have been shown to reduce relapse

Substances that belong to the “depressant” class

ETOH
cannabis
opioids
inhalants
benzodiazepines

Which of the following statements regarding the mechanism of action of vareniciline is accurate

It has activity at Alph 4 beta 2 subtype of the nicotinic receptor
It is effective for nicotine/tobacco cessation

A 50 year old man with chronic alcohol use disorder ran out of ETOH and drank methyl alcohol from the factory where he is employed 18 hours ago. He presents at ER with encephalopathy, ataxia, seizures, and bilateral vision loss. Imagineer reveals an intracranial hemorrhage. What is the likely location of his hemorrhage

Putamen (formic acide accumulates in this region)

Long term survivors of methyl alcohol intoxication may experience

Parkinsonism

Chronic exposure to methyl alcohol may result in

axonal polyneuropathy

A 29 year old patient is having marital struggles and multiple recent arguments with her husband presents to the ED complaining of R sided weakness. A neurological evaluation, including imaging, reveals no identifiable diagnosis. Psychiatric consultation is requested. After 2 days, the patient’s weakness has subsided and she is discharged home. Which of the following is a positive prognostic factor for her diagnosis

Short duration of illness

A 38 year old woman presents to the psychiatry clinic wearing a wig. She has no eyelashes or eyebrows. Which behavioral therapy has been shown to be particularly helpful for this patient’s self-inflicted condition?

Habit Reversal

Treatment for trichollomania

Medication and behavioral therapy (habit reversal – apply barriers such as gloves or hat)

A 19 year old male presents with facial numbness and bilateral vision loss. He reports tingling, cold feet with bluish discoloration. His symptoms most likely resulted from inhalation of which substance.

n-Hexane (household glues)

Repeated glue inhalation may cause ______ which has a similar presentation to ______

glue sniffer neuropathy
Guillain Barre Syndrome

Which antidepressant with a tetracyclic structure that works by antagonism of presynaptic alpha 2 agonists and post-synaptic serotonergic receptors is of interest for a psychiatrist looking to prescribe pharmacologic treatment to a patient with chronic alcohol use disorder with a major depressive episode?

Mirtazapine

This would not show on the toxicology screen and is a drug of abuse, particularly in pubescent boys, that could cause a period of unconsciousness

Nitrites (present in liquid incense)

onanism

masturbation

Amnesic disorders are secondary syndromes caused by

systemic medical or primary cerebral diseases
substance use disorders
medication adverse effects

In contrast to transient global amnesia, dissociative amnesia typically does not involve

deficits in learning and recalling new information

Dissociative disorders are often associated with

emotionally stressful life events involving the money, legal systems, or troubled events

What is the lifetime suicide-attempt rate in patients with body dysmorphic disorder

22-24%

Wernike’s encephalopathy triad

confusion
ataxia
ophthalmolopegia

What type amnesia is classically associated with confabulation do Korsakoff disease patients have?

anterograde and retrograde amnesia

Xerostomia

dry mouth caused by reduction or absent flow of saliva

Icterus

jaundice

According to a recent study, the volumes of which brain structures are significantly smaller in patients with dissociative identity disorder versus healthy subjects?

Hippocampus and amygdala

In using the Structured Interview of Reported Symptoms (SIRS) test, when would a patient be considered to be malingering?

They score in the definitive range in 1 primary subscale or probable range in 3 sub scales

A 48 year old pt admitted with profuse sweating, diarrhea, abdominal cramps, vomiting, yawning, muscle aches, restlessness, and insomnia for 1 day. Physical exam reveals BP 155/105 mmHg and a temp of 100 degrees F. The patient is cold with muscle twitches and dilated pupils. She is highly motivated to detoxify. Which medication is the best choice for detox?

Methadone

Which of the following is useful in treating the autonomic effects of opiate withdrawal

Clonidine

Which of the following is useful for treating hyperthermia in opiate withdrawal

cold blankets
ice packs

Which of the following is useful for treating seizures in cocaine overdose

benzodiazepine

Which of the following is useful for treating malignant hypertension in cocaine overdose

IV phentolamine

Which 2 medications are useful for treating agitation in cocaine overdose?

lorazepam
haloperidol

What is the most common type of psychiatric condition in an emergency room

Substance-related disorders

“You have been through so much recently but you remain certain that alcohol is not a problem for you” This is an example of a

reflective statement

“What do you feel about all this?” is an example of

open-ended question

“You have done a phenomenal job staying sober in the past.” is an example of

affirmation

“let me see if I understand you. It sounds like you’ve been trying to stop using for some time but are on the fence right now about stopping. Is that about right?” is an example of

Summary Statement

OARS stands for

Open-ended questions
Affirmations
Reflective listening
Summaries

4 core techniques for motivational interviewing include

expression of empathy
rolling with the resistance
developing discrepancy
supporting self-efficacy

Successful malingerers are more likely to have which characteristic compared to unsuccessful maligerers

They are more likely to report fewer symptoms

Pt got Tdap 10 years ago, what is she due for

Td booster

Which type of group treatment affects the largest number of substance abuse patients

self-help groups

inability to recall autobiographical information and apparently purposeful travel associated with amnesia. Onset sudden, memory loss may be refractory

dissociative amnesia with dissociative fugue

Pt often feel detached from their body, as if they are watching from outside their body. There is no associated amnesia or change in personality or identity

Depersonalization/derealization disorder

Temporary episode of memory loss which can not be associated with a neurological cause. there is no associated change in identity. Onset is sudden, and the person is unable to recall who they are, how they got there, or what they were doing

Transient global amnesia

Factitious disorder involves

deceptively reporting false symptoms to assume the “sick role”

Dangerous dose of methadone may present as

drowsiness
motor impairment
mitosis
nausea

Safest initial methadone dose

10-20mg given in single dose

A dose of ____ methadone can be given if withdrawal symptoms persist an hour after the initial dose

5mg

Piloerection

goose bumps

On withdrawal for methadone, signs of insomnia, fatigue, irritability and anxiety may last for

weeks to months

Typical methadone maintenance dose

60-100mg daily

The most difficult period is when tapering methadone maintenance at doses

<25mg

Which of the following disorders does not fall into the category of somatic symptom and related disorders?

Somatization disorder

Examples of Somatic symptom and related disorders

somatic symptom disorder
illness anxiety disorder
conversion disorder
factitious disorder

Approximately which percentage of patients with chronic alcohol use disorder have at least 2 parents with chronic alcohol use disorder

30%

Earlier symptoms of GHB withdrawal

Anxiety
Insomnia
Tremor
confusion
nausea
vomiting

Early symptoms of MDMA withdrawal

Fatigue
dysphoria
depression with loss of appetite
trouble concentrating

Although results vary based on age, gender and drinking pattern of patient, which 2 biomarkers will provide NP up to 90% sensitivity for heavy, chronic alcohol usage?

Glutamyltransferase (GGT) & carbohydrate deficient transferrin (CDT)

Blood test that detects heavy alcohol use (5+ or more drinks a day) over the past two weeks

CDT

45 year old male brought to ER by son in agitated and disoriented state after tonic clonic convulsions. BP 170/105, pulse 125, RR 15, Temp 99, What is his most likely dx?

Alcohol withdrawal

Can resemble manic episode with euphoria, impulsive behavior, aggression, dystonia, heightened self-esteem, mydriasis, increased vitals, hyperthermia, cardiac conduction abnormalities

Cocaine intoxication

Common symptoms of include a subjectively altered sense of time, subjective difficulty in vividly recalling past memories, vague somatic symptoms, and fear of irreversible brain damage

Depersonalization/derealization disorder

There is an association with childhood interpersonal trauma, and it can be precipitated by severe stress or illicit drug use (most notably hallucinogens, ketamine, MDMA and salvia)

Depersonalization/derealization disorder

What drug can precipitate new onset panic attacks and depersonalization/derealization disorder at the same time

Marijuana

Average age of onset of depersonalization/derealization disorder

16

Depersonalization disorder is _____ more common in woman than men

2-4x

What pneumococcal vaccine is indicated for all adults over 65 years old

PPSV23

What pneumococcal vaccine is indicated for adults <65 with chronic lung conditions, chronic CV disease, DM II chronic renal disease, chronic liver diseases, chronic alcohol use disorder as well as immunocompromising syndrome

PPSV13

People who display factitious disorder by proxy are more likely to have which condition

Personality disorder

Diarrhea, lacrimation and runny nose are mainly associated with

opioid withdrawal

Beta blocker given when pt has cocaine intoxication could possibly cause?

unopposed alpha stimulation

May have recurrent gaps in recall of everyday events, not just of traumatic experiences.

Disassociative identity disorder

What is Quetiapine FDA approved for

bipolar depression
schizophrenia
acute mania

What is Aripiprazole FDA approved for

Agitation in autism, bipolar depression, bipolar mania, schizophrenia

What is risperidal FDA approved for

Schizophrenia, acute/mixed mania, Autism (5-16 years), bipolar maintenance

What is Olanzapine FDA approved for?

Schizophrenia,
bipolar mania and depression
agitation

What is Clozapine FDA approved for?

schizophrenia (especially tx resistant)
schizoaffective

What is asenapine FDA approved for?

Schizophrenia
acute/mixed mania

What is Lurasidone approved for?

schizophrenia

What is ziprasidone approved for

schizophrenia
acute psychosis
agitation
bipolar disorder
mania

What is iloperidone FDA approved for?

schizophrenia

A 40 year old F is taking theophylline for her asthma. She is known to have a prolonged hx of tonic-clonic seizures. However, you remember from your studies that CYP450 participates in theophylline metabolism, and that certain anti-epileptic affect CYP450 activity. Which adjustment is likely to be warranted in this patient?

Give carbamazepine
Consider increasing the theophylline dose in the future

Theophylline

bronchodilator with a very narrow therapeutic index
Metabolized by CYP450

Valproic acid ________ CYP450 causing theophylline to _______

inhibits
accumulate (need lower dose)

Phenobarbital __________ CYP 450 causing theophylline to _______

induces
clear faster (need higher dose)

Carbamazepine __________ causing theophylline_______

induces CYP450
to clear faster (need higher dose)

Phenytoin ________ CYP450 causing theophylline to ________

induces
clear faster (need higher dose)

After stopping diazepam in pt who is a chronic user

Monitor patient for a week (withdrawal seizures occur about 5-7 days after drug is stopped)
Ideally should be gradually tapered at max rate of 10% per day.

You will not see any signs of withdrawal of diazepam until

5-7 days due to long half-life

Which of the following statements is correct concerning clozapine

TD is not commonly associated with clozapine compared to other antipsychotics

Agranulacytosis is the most significant adverse effect of

clozapine

If agranulocytosis occurs in a pt on clozapine, it will more likely happen

in first 6 months of tx
to woman
Ashkenazi Jewish descent
to those of advanced age

______ of patients on dosage of _______ of clozapine will have seizures

4-6%
>600mg/day

SSRI’s and coagulation

increase risk of bleeding d/t interactions with serotonergic receptors on platelets

What SSRI’s are most likely to have gastrointestinal side effects

sertraline
fluvoxamine

CNS activation (anxiety, restlessness, tremor, and insomnia) occur most commonly with this SSRI

fluoxetine

SIADH commonly occurs with this SSRI

fluoxetine

Sedation occurs most commonly with this SSRI

paroxetine

Which of the following statements is true regarding Gabapentin?

It does not interact with anticonvulsants
Somnolence and dizziness are the common side effects and may lead to discontinuation

How does trazodone cause activation of the CNS?

Metabolite activity as well as inhibition of serotonin reuptake in presynaptic receptors

What is correct regarding methylphenidate (MPH)

patients on transdermal MPH require lower doses than oral extended-release (transdermal does not make first pass in liver)

The ratio of transdermal MPH to oral extended release MPH is

10:15mg

MPH is currently manufactured as a mixture of

d and l-threo enantiomers
(d-threo = active form )

Clozapine
-Anticholinergic Effect
-Sedation
-Extrapyramidal Effects

High
High
No

Quetiapine
-Anticholinergic Effect
-Sedation
-Extrapyramidal Effects

Low
High
Low

Haloperidol
-Anticholinergic Effect
-Sedation
-Extrapyramidal Effects

Low
Low
High

Olanzapine
-Anticholinergic Effect
-Sedation
-Extrapyramidal Effects

Low
Mid
Low

Risperidal (Risperidone)
-Anticholinergic Effect
-Sedation
-Extrapyramidal Effects

Low
Low
Low

Ziprasidone (Geodon)
-Anticholinergic Effect
-Sedation
-Extrapyramidal Effects

Low
Low
Low

Aripiprazole (Abilify)
-Anticholinergic Effect
-Sedation
-Extrapyramidal Effects

Low
Low
Low

Paliperidone (Invega)
-Anticholinergic Effect
-Sedation
-Extrapyramidal Effects

Low
Low
Low

Chlorpromazine (Thorazine)
-Anticholinergic Effect
-Sedation
-Extrapyramidal Effects

High
High
Low

What is the mechanism of action of memantine

NMDA receptor antagonist (blocks excessive stimulation of glutamate receptors but does not affect normal transmitter activity)

Which of the following classes of antidepressant drugs includes mirtazapine

Tetracyclic antidepressants
(noradrenergic and specific serotonergic antidepressant (NaSSA)

Mechanism of action of Mirtazapine

blocks presynaptic alpha-2 andrenergic receptor ->
5-HT-1A activation and increased dopamine release

Which FDA established category indicates a potential risk of a drug to cause birth defects in pregancy because there are no adequate and well-controlled studies to show the drug to cause birth defects in pregnant women?

Category B

Which of the following statements about carbamazepine is correct

Neutropenia occurs often

Serum levels of carbamazepine equilibrate after a fixed daily dose is given for

3-5 days

Metabolism of carbamazepine occurs through the

P-450 3A4 isoenzyme

Carbamazepine mechanism of action is

blockade of sodium gated channels

Food intake _____ the absorption of carbamazepine

increases

Lab draws with carbamazepine

CBC 2 weeks, then at 6 weeks, then q6 months thereafter

Which statement regarding bupropion is correct

there is no withdrawal syndrome associated with abrupt discontinuation

Absolute contraindications for bupropion

seizure disorder/hx
eating disorder
head trauma
alcohol abuse

______ is a dopamine/NE reuptake inhibitor

bupropion

Side effect of nefazodone

priapism

Side effect of MAOIs

pyridoxine deficiency

Which statement regarding side effects of valproic acid is correct

Pancreatitis occurs in 1/6000 pts

Which EPS side effect is most commonly observed with clozapine use?

Akathisia (3%) due to dopamine antagonism

The action on _________ is responsible for pseudoparkinsonism or dystonia

nigrostriatal system

Half life of triazolam

3-6 hours (peak blood level in 20 minutes)

Half life of Diazepam (Valium)

40 hours

Half life of Flurazepam

40 hours

A woman who has been on a long-term treatment for anxiety no longer feels relief after taking her Rx medication at max dose for the past dose? What explains her tolerance

Saturated binding sites

Which amphetamine isomer is most rapidly eliminated from the body

Dextro-isomer in acidified urine

Which statement regarding tricyclic antidepressants (TCA) and anticholinergic side effects is true?

TCAs can be dangerous to patients with pre-existing glaucoma and lead narrow-angle glaucoma

Examples of anticholinergic effects

dry mouth
blurry vision
constipation
tachycardia
delirium

TCA’s can cause sedation and hypotension d/t

alpha-1 receptor blockade in blood vessels

Anticholinergic effects result from the

blockade of muscarinic cholinergic receptors

What enzymes are responsible for metabolizing 90% of drugs processed in liver

1A2
2C9
2C19
2D6
3A4
3A5

Example of drugs to avoid when on bupropion

Clozapine
Theophylline
Clomipramine
(d/t inc risk seizures)

Which of the following benzodiazepines is most rapidly absorbed when taken orally?

diazepam

Which statement is true of the Minnesota Multiphasic Personality Inventory (MMPI)

It is a broad-based measure of personality traits and psychopathology

It test cognitive ability and intelligence

Stanford-Binet Intelligence Scale

It measures cognitive impariment

MMSE

It measures a person’s emotional functioning

Rorschach test (“inkblot” test)

Which of the following statements about lithium-induced hypothyroidism is correct

occurs more commonly in woman

Hypothyroidism is not a contraindication for

lithium treatment (can be treated with levothyroxine)

Hypothyroidism in pt receiving lithium is associated with

rapid cycling
increased risk of developing depression

Blood tests with lithium

Thyroid function test (TFT) at least once in the first 6 months of treatment
BMP (renal function) every 3 months
After first 6 months, LFT, TFT q 6-12 months

Lithium induced hypothyroidism generally occurs within the first

6-18 months

Half life of this hypnotic is 1-3 hours

ramelton

Half life of this hypnotic is 1-2 hours

zaleplon

Half life of these hypnotics is 2-4 hours

triazolam
zolpidem

Antidepressant medication with significant anticholinergic side effects do not need to be avoided in patients with which comorbidity

constipation (can be managed by bulk-forming laxatives and hydration)

CYP450 substrate of Alprazolam

3A4

CYP450 substrate of Citalopram

2D6

CYP450 substrate of Codeine

2D6

CYP450 substrate of Caffeine

1A2

Acute alcohol consumption _______ CYP450, but chronic use leads to __________ CYP450

inhibits
induces (break down faster – sub therapeutic level)

Monitoring of liver function tests every 6-12 months is required with which of the following mood stabilizers

valproic acid

Side effects of carbamazepine

sedation
dizziness
fatigue
nausea
ataxia
agranulocytosis
aplastic anemia

side effects of lithium

Tremor
polyuria
polydipsia
weight gain
cognitive slowing
hypothyroidism
decreased creatinine clearance

Side effects of lamotrigine

benign rash in 10% of people
Steven’s Johnson syndrome in <1%

Withdrawal syndrome following abrupt discontinuation of TCA results from

cholinergic rebound

Quinidine and TCA both

prolong QT intervals

Adverse effects of TCA overdose

severe sedation
hypotension
anticholinergic delirium

Although lorazepam has a relatively short half life, it exerts sustained action via which mechanism

lower lipophilicity

Being less _______ produces clinical effects more slowly, but may provide more sustained relief, as the drug comes out of the brain slowly

lipophilic

Gabapentin overdose is associated with

sedation
ataxia
diplopia

Which one more commonly causes weight gain: gabapentin or valproic acid

valproic acid

Adverse effect of Topiramate

kidney stone formation (especially after CYP450 activity inhibited d/t acute alcohol ingestion)

St. John’s wort effect on Topiramate

increases clearance d/t induced CYP450 activity

What medications can cause a hypertensive crisis when used with MAOI?

meperidine
epinephrine
local anesthetics (containing sympathomimetics)
decongestants

Pt on clozaril and is concerned that PCP prescribed pt this for her psoriasis

methotrexate (inc risk leukopenia)

What antidepressant has the greatest evidence of beneficial effects in patients with depressed mood and eating disorders?

Fluoxetine

In CATIE, patients were assigned to treatment with

olanzapine
perphenzine
quetiapine
risperidone
or (later) ziprasidone

A 55 year old F takes metoprolol to control her HTN. She has toenail fungus and decides to take her husband’s Rx for terbinafine. What is the most likely outcome

hypotension (unRx med is 2D6 potent inhibitor leading to elevated levels of the HTN med)

Specific Gravity 1.045 indicates

fluid volume deficit
dehydration
SIADH
CHF
Acidosis
Prerenal failure

By what age is the process of absorption and transportation of drugs comparable between children and adults

4 months

ECT deliberately induces what type of phenomenon

generalized tonic-clonic seizures

What are 2 common side effects of fluoxetine

diarrhea
cramps

All women of childbearing age treated with valproic acid should also receive which of the following vitamin supplements

Folate (B-9)

Which of the following medications has the most evidence in its effectiveness for treating drug induced psychosis in patients with Parkinson’s disease

Clozapine
(Olanzapine is a close second)

Which of the following antipsychotic medications has the fewest anticholinergic side effects?

Haloperidol

In recent years, ketamine has been used in many trials for treatment-resistant depression (TRD). Which of the following drugs shares a similar action to ketamine?

Phencyclidine (PCP)

Sedating antidepressants

trazodone
nefazodone
imipramine
amitriptyline
trimipramine
doxepin
mirtazapine
amoxapine
paroxetine

TCA that tends to be energizing instead of sedating and is sometimes used for narcolepsy to achieve a wakefulness promoting effect

protriptyline

Which statement is correct regarding the side effects of risperidone

When taken at high doses, it has a higher rate of tar dive dyskinesia than other atypical antipsychotics

The class effects of risperidone inclued

alpha 1 blockade (dizziness, sedation, hypotension(orthostatic))
D2 blockade (motor)
muscarinic 1 blockade (dry mouth, constipation, sedation)
histamine 1 blockade (sedation and possible weight gain)

Black box warning with risperidal

increased death rates in patients with dementia

Initial side effects of risperidal

dizziness
orthostatic hypotension
headache

Risperidal has the highest risk of ______________ compared to all other antipsychotics

hyperprolactinemia

The risk of diabetes and weight gain are highest in these 2 antipsychotics

olanzapine
clozapine

Drug-drug interaction of fluoxetine and TCA

increased cardiac risk

Drug-drug interaction of fluoxetine and thioridazine

increased cardiac risk

Drug-drug interaction of fluoxetine and benzodiazepine

increased sedation

Drug-drug interaction of fluoxetine and carbamazepine

increased carbamazepine levels (toxicity)

Drug-drug interaction of fluoxetine and buproprion

increased seizure risk

Drug-drug interaction of fluoxetine and phenytoin

increased phenytoin toxicity

Drug-drug interaction of fluoxetine and warfarin

warfarin toxicity

2 side effects of zolpidem

daytime sleepiness
fatigue

Side effect of eszopiclone (Lunesta)

daytime sleepiness
fatigue
taste disturbance

Buspirone side effects

dizziness, headache, nausea
no long term effects
no physiological dependence or withdrawal syndrome

Most common negative side effect of zolpidem

sleep-associated disorders

short acting sedative sleep agent that is associated with test disturbance

Eszopiclone

Primary overdose symptoms with clozapine include

delirium
lethargy
tachycardia
hypotension
respiratory failure

What is the most common side effect of galantamine?

GI disturbances (diarrhea, nausea, vomiting decreases over time)

Class of drug of galantamine

cholinesterase inhibitor

Because cholinesterase inhibitors cause action in the parasympathetic nervous system they can cause

bradycardia
hypotension
hypersecretion
bronchoconstriction
GI tract hypermotility
decreased intraoccular pressure

Priapism is associated with which drug

trazodone

At 45mg mirtazapine, what can happen?

increased serum cholesterol inducing cases of acute pancreatitis and diabetic ketoacidosis.

Mood stabilizers like lithium in pregnancy

stopped until 2nd trimester to avoid the time of highest teratogenic risk.

In neonates, high potency antipsychotics

are least likely to cause anticholinergic, antihistaminic, or hypotensive effects (unlike low potency medications)

What medication has the most evidence supporting its use in bipolar disorder and schizophrenia in pregnant women

haloperidol

effective and safe treatment for severe mania in pregnancy

ECT

Neonates can show signs of extrapyramidal side effects when

a high potency antipsychotic is used near term

When switching from an SSRI or SNRI to an MAOI, which of the following medications requires a 5-6 washout period?

Fluoxetine

Besides Fluoxetine, all other SSRI, SNRIs, and TCAs require a standard ___________ washout period when switching to an MAOI

2 week

What medication demonstrates histaminergic mechanism in addition to increasing dopamine binding to the dopamine transporter to increase wakefulness

modafinil

Modafinil increases activity in the

tuberomammilary nucleus
stratum
cingulate cortex (at higher doses)

Humans with narcolepsy have decreased levels of

histamine in the CSF

Modafinil ________ to the dopamine transporter

weakly binds (inhibiting reuptake of dopamine)

Dopaminergic mechanisms that are affected by traditional stimulants, such as amphetamine, are not thought to account for the ___________ induced by modafinil

wakefulness

What is the mechanism of action of topiramate?

inhibits sodium channels
enhances GABA-a receptors
antagonizes kainite and AMPA (glutamate) receptors

Carbamazepine mechanism of action

Inhibits voltage-gated sodium channel activation (doesn’t block pores).
inhibits pre-synaptic sodium channels

Depakote mechanism of action

increases GABA

Lamotrigine mechanism of action

inhibits glutamate
inhibits voltage-gated sodium channels

centrally acting, reversible acetylcholinesterase inhibitor used in tx of Alzheimer’s dementia

Donepezil (Aricept)

Why is Donepezil contraindicated in sick sinus syndrome?

Can cause heart block and symptomatic bradycardia

Flushing is caused by

excessive dilation of cutaneous arteries caused by muscarinic blockade (eg atropine)

Rash is less common in these drugs, but patients on these 2 drugs are at risk for Steven’s Johnson syndrome

oxcarbamazepine
carbamazepine

A low ______ value can occur in isolation with oxcarbazepine

T4

What mood stabilizer is associated with aplastic anemia and other blood dycrasias in 1 in 10,000-150,000 treated pts?

Oxcarbazepine

Carbamazepine overdose is associated with

high grade AV block
CNS depression

Oxcarbazepine overdose is associated with

CNS depression

Adjuvant therapy in acute mania to treat hyperkinesis, agitation and insomnia

benzodiazepine

Pt’s with hx of ADHD in childhood may be at particular risk for medication induced agitation with this medication

benzodiazepine

What is the initial medication dosage for treatment of uncomplicated depression:
sertraline ____

50mg (increase q 2 weeks to max 200)

What is the initial medication dosage for treatment of uncomplicated depression:
Fluoxetine ______

10mg (maintain for 3 weeks, max benefit at 20-40mg)

Typical maintenance dose of fluvoxamine

100-200mg/day (short half life)

Typical maintenance dose of fluoxetine is

20-40mg/day

Doses of fluoxetine ______ have more side effects and have not been effective

>=60mg

A maximum of ______ is recommended for initial treatment with paroxetine

50mg

Which of the following findings is expected in a patient who is an ultra-rapid metabolizer

High urinary concentration of the drug’s metabolite
Low plasma concentration of the drug

Which statement correctly describes the inducer/inhibitor pharmacology of sertraline and tramadol when a patient is taking both medications?

Sertraline at 200mg is a potent inhibitor of 2D6 and increases levels of tramadol (increased risk seizures)

Inhibitors reduce the metabolism of ______ leading to elevated levels of ___________

medications/substrates
medications/substrates

To have lower levels of tramadol, a patient must take a CYP2D6 _______

inducer

What medications selectively inhibit the reuptake of both serotonin and norepinephrine

SNRIs:
duloxetine
venlafaxine
desvenlafaxine

Antidepressant with relatively weak reuptake inhibitor of dopamine and norepinephrine

Bupoprion

Noradrenergic and specific serotonergic antidepressant (NaSSA) that acts by antagonizing the adrenergic alpha-2 autoreceptors and alpha2-heteroreceptors, as well as by blocking 5-HT2 and 5-HT3 receptors

mirtazapine

Mechanism of action of ramelteon (Rozerem)

melatonin MT1 and MT2 receptor agonist

GABA-a receptor agonists are better known as

benzodiazepine receptor agonists

Side effects of chlorpromazine

photosensitivity
very rarely Neuroleptic malignant syndrome (NMS)

Cholestatic jaundice occurs at a rate of 0.1% with

chlorpromazine

What medication taken in large doses can cause mild pigmentary changes in the retina

chlorpromazine

This medication_______ (dosed at _____) is associated with irreversible pigmentary retinopathy, and blindness can occur

Chlorpromazine >800mg/day

What lab would you order on a reproductive aged female patient prior to beginning depakote?

human chorionic gonadotropin (HCG)

Depakote can cause this in pregnant woman

major congenital malformation, particularly neural tube defects

Cyclophosphamide (Cytoxan) and carbamazepine

auto-induce their own respective metabolisms

Theophylline, ifosfamide and aminogluthimide

induce their own metabolisms

Valproic acid and isoniazid are both

inhibitors

Chronic doses of ethanol = ______, but in acute, binge doses = _______

induction
inhibition

Ziprasidone may cause

weight loss

In which neuropsychiatric condition are levels of coenzyme Q10 lower than normal

Parkinson’s disease

Initial studies show that supplementation of CoQ10

lower BP
improve CHF

What medication requires monitoring of symptoms like bruising and bleeding?

Carbamazepine

Discontinuation of Valproate is recommended if

signs of bleeding develop (dose-related thrombocytopenia)

Carbamazepine can cause suppression of

bone marrow (agranulocytosis, aplastic anemia, thrombocytopenia)

When used with lamictal, depakote dose should be

reduced in half

May be best for mixed episodes, more effective in mania vs depression

depakote

Blockade of voltage gated sodium channels, decreases glutamate release

cabamazepine
lamictal

Use reserved for treatment resistant bipolar disorder due to multiple drug-drug interactions

carbamazepine

Unique side effects of aplastic anemia, Stevens Johnson syndrome and SIADH

carbamazepine

Unique side effect of hyponatremia (especially with SSRI). No association with SJS or aplastic anemia. CYP3A4 inducer

oxcarbazepine

Unusual side effect of metabolic acidosis, kidney stones

topiramate

Unusual rare side effect of hepatoxicity and pacreatitis

depakote

Unusual side effect of thyroid dysfunction, chronic renal insufficiency, decreased cardiac conduction efficiency, hypercalcemia, mild leukocytosis

Lithium

Linezolid, dextromethorphan, meperidine, propoxyphene, methadone, tramadol and high-dose sumatriptan all have this in common

greater risk of serotonin syndrome when added to an SSRI.

All triptan medications used for migraine treatment will cause cause a significant increase in ________ at higher levels

serotonin

Smoking and clozapine

decreases clozapine level

Tobacco is a _____________ enzymes

potent inducer of 1A2

Daily smoking causes antipsychotics to be ________

cleared more quickly necessitating higher doses

St. John’s wort is an ________ agent for _______

inducer
3A4
3A5

Substrates affected by St. John’s wort include

amprazolam
amlodipine
atorvastatin
diazepam
cyclosporine
serotinergic meds

Cruciferous vegetables induces the metabolism of _________

1A2

Increasing consumption of cruciferous vegetable would _________ clozapine levels

lower (need higher dose to decrease risk for psychosis)

For the ER form of divalproex sodium, what is the correct dosing adjustment compared with regular formula

increase ER dose by 15%

What is an appropriate loading dose of valproic acid for a hospitalized patient with significant manic symptoms?

20-30mg/kg

Ventricular arrhythmia and seizures can be the result from ______ overdose

TCA

Cardiac conduction defects are most severe with ______ overdose

desipramine

TCAs are structurally similar to

class I arrhythmias (20% proarrythmic & 20% with preexisting conduction disturbances have cardiac complications when taking)

Tyramine-containing foods should be avoided in pts treated with

MAOI

Stevens-Johnson syndrome is associated with

Lamictal (Lamotrigine)

SIADH, leading to hyponatremia, has been associated with

carbamazepine (Tegretol)

Primary consideration when dosing fluoxetine for 9 year old boy

height and weight

Adult values for the cytochrome 450 enzymes are generally achieved by what age?

1 years old

Psychotropic medications are not generally prescribed to children under the age of

5 years

Fluoxetine is FDA-approved in the treatment of depression for what ages

ages 8 years and older

Monitor ADHD patients on stimulants for

palpitations
syncope
CP
vital signs including high blood pressure

What supplement can cause easy bruising?

Ginkgo biloba

Gingko biloba should be avoid with

antiplatelet and anticoagulant medications

Supplement contraindicated when taking warfarin

gingko biloba

Anti-inflammatory and immune enhancing supplement with no known drug interactions

Echinacea purpurea

Another name for garlic

Allium sativum

Why should garlic be used cautiously with warfarin?

anti-platelet effects

Which SSRI has been associated with hyponatremia and SIADH, particularly in elderly females

sertraline

Appropriate ED treatment for panic disorder

benzodiazepine

Recurrent episodes with at least four of the following symptoms:
–trembling, diaphoresis, nausea
–CP/palpations/dyspnea/choking sensation
–intense anxiety/fear of dying or losing control
–derealization
–depersonalization
–dizziness
–paresthesias
–anxiety about additional attacks
–avoidance behavior

panic disorder

First line tx of panic disorder

SSRI (fluoxetine, paroxetine, sertraline)
or SNRI (venlafaxine)

MAO-B inhibitor with FDA indication for depression, Parkinson’s disease and major depression

selegiline

pheochromocytoma

a benign tumor of the adrenal medulla that causes the gland to produce excess epinephrine
It is associated with episodes of tachycardia, HA, diaphoresis, and paroxysmal or sustained HTN

SSRI-induced sexual dysfunction occurs in

men mostly (tends not to dissipate with ongoing tx but does dissipate on dc tx)

L-carnitine is effective in the treatment of methamphetamine toxicity via which mechanism

preventing formation of free radicals and peroxynitrite in neurons
decreases free radicals in the neuron
needed to transport long-chain fatty acids into the mitochondria for fatty acid oxidation

Which cytochrome P450 enzyme is predominantly present at but levels fall quickly and are undetectable in most adults?

3A7

Last P450 cytochrome to develop

1A2

P450 enzymes present in newborns at very low concentration but surge quickly to about 20-30% of adult values within the first week of life

3A4
2D6
2C9/19

Erythromycin _______ the metabolism of drugs

decreases (inhibits)

sick-faces.com/g is a mnemonic to remember

inhibitors

list drugs on the sick-faces.com/g mnemonic of inhibitors

sodium valproate
isoniazid
cimetidine
ketoconazole
fluconazole
alcohol binge
chloramphenicol
erythromycin
sulfonamides
ciprofloxacin
omeprazole
metronidazole
grapefruit juice

smoking, griseofulvin and carbamazepine are all

inducer

What birth defect has been associated with lithium use in pregnancy (only psychoactive drug associated with this)

Ebstein’s anomaly

PCOS is associate with ________ use in pregnancy

valproic acid

Narcolepsy with cataplexy and excessive daytime sleepiness FDA approved treatment

sodium oxybate (Xyrem)

What antidepressants can reduce cataplexy attacks

clomipramine
fluoxetine
duloxetine
venlafaxine

<1% is excreted unchanged in feces and urine

trazodone

Trazodone is about ________ protein bound

89-95%

Trazodone’s peak plasma level occurs after

1 hour when taken on empty stomach
2 hours when taken on full stomach

What inhibits trazodone’s clearance

3A4 inhibitors (ritonavir, indinavir, ketoconazole)

When a pt taking lamotrigine gets a rash accompanied by _________ , s/he should go to the ER immediately

systemic symptoms such as fever or discomfort in mouth, eye, or bladder

The risk of rash with lamotrigine is highest with

rapid titration and around dose increases

Which medication has a side effect leading to the highest suicide risk in female patients?

valproic acid (can cause PCOS-10% every year)

Women with PCOS were far more likely than women without PCOS to have

mood and anxiety symptoms
7x more likely to make suicide attempt

What is the mechanism of action of sumatriptan

5HT1D and 5HT1B agonist

What is the mechanism of action of aripiprazole

5HT2C agonist
weak partial agonist 5HT7

Pts with hx of seizure d/o are more safely treated for depression with ______ than with ____-

SSRI or venlafaxine
TCA or bupropion

What mood stabilizer can cause hypernatremia

Lithium

What medication frequently induces nephrogenic diabetes insipidus

lithium

How does lithium cause nephrogenic diabetes insipidus

it competes with receptors for the antidiuretic hormone in the kidney, increases water output in urine, causing hypernatremia

What attenuates lithium induced diabetes insipidus

amiloride

Benzodiazepine anxiolytic subclasses are divided according to their chemical structures. What does this affect?

half-life of the drug in body

Half-life of diazepam

40 hours (oxidized in liver)

Temazepam, lorazepam, and oxazepam are metabolized by ______ and cleared by ________

glucuronidation (hepatic dysfunction not affected)
kidney

Besides hypotension, tachycardia, and weight gain, the side effect of clozapine that is most common

hypersalivation/drooling (10-40%)

Medication for sleep-onset insomnia that is not habit forming/no abuse potential

Ramelteon (melatonin agonist)

Pt on Coumadin recently prescribed fluconazole. What finding would you expect at her next INR blood test

elevated

Blocks voltage-dependent sodium channels and increases gamma aminobutyric acid (GABA) in the brain. Both inhibits GABA metabolism and increases its synthesis

Valproic Acid

mechanism of action of modafinil

1A2 inducer
3A4 substrate and inducer

Example of CYP1A2 inducers

modafinil
charcoal broiled beef
tobacco
marijuana
omepraxole
cruciferous vegetables

List 6 CYP2C19 inhibitors

cimetidine
ketoconazole
lansoprazole
modafinil
omeprazole
topiramate

CYP2C19 inducers

carbamazepine, phenytoin, rifampin, prednisone

CYPD6 inhibitor

bupropion
citalopram
duloxetine
fluoxetine
fluvoxamine
paroxetine
cimetedine
doxepin
haloperidol
methadone

CYP2D6 inducer

dexamethasone
rifampacin

CYP1A2 inhibitor

Fluvoxamine
Paroxetine
Amiodarone
Fluroquinolones
Cimetedine

CYP1A2 inducers

insulin
modafinil (also 3A4 inducer)
omeprazole
phenobarbital (also 3A4 inducer)
rifampicin (also 3A4 inducer)

CYP2B6 inhibitors

Ticlopidine

CYPB6 inducers

Phenobarbital
Rifampicin

CYP2C8 inhibitors

gemfibrozil
giltazones

CYP2C8 inducers

1. Carbamazepine (Tegretol)
2. Phenobarbitol
3. Phenytoin (Dilantin)
4. Rifampin (Rifadin)

CYP2C9 inhibitors

Amiodarone
Fluconazole
isonazid
sulfa drugs
trimethoprim

CYPC19 inducers

Phenobarbital
Rifampicin

Bupoprion is a _______ substrate

2D6

Methadone is a _______ substrate

2D6 (and inhibitor)

Fluoxetine is a ______ substrate

2C9
2C19
2D6 and inhibitor
(3A4 inhibitor)

Phenytoin is a _______ substrate

2C9
2C19
(3A4 inducer)

Sertraline is a _______ substrate

2C9
2C19
2D6

Fluoxetine is a _______ substrate

2C9
2C19
2D6 and inhibitor
(3A4 inhibitor)

Fluvoxamine is a ______ substrate

2C9
1A2
2C19
2D6 and inhibitor

Amitriptyline is a _____ substrate

1A2
2C19

Clozapine is a _____ substrate

1A2

Duloxetine is a _____ substrate

1A2
2D6 and inhibitor

Haloperidal is a _____ substrate

1A2
2D6 (and inhibitor)

Imipramine is a _____ substrate

1A2
2C19

Mirazapine is a _____ substrate

1A2
2D6

Ondanseteron is a _____ substrate

1A2

Warfarin is a _____ substrate

1A2
2C9

Olanzepine is a _____ substrate

1A2
2D6

Citalopram is a _______ substrate

2C19
2D6 (and inhibitor)

Diazepam is a _____ substrate

2C19

escitalopram is a _____ substrate

2C19
2D6

Paroxetine is a _______ substrate

2D6 and inhibitor

Amphetamine is a _________ substrate

2D6

Atomoxetine is a _______ substrate

2D6

Clomipramine is a _________ substrate

2D6

Risperidal is a _____ substrate

2D6

Venlafaxine is a _____ substrate

2D6

Buspirone is a _______ substrate

3A4

St. John’s wort is a _______ inducer

3A4

Oxcarbazepine and carbazepine are __________ inducers

3A4

Grapefruit juice is a ______ inhibitor

3A4

Clonazepam (Klonopin) is a ______ substrate

3A4

Xanax is a ______ substrate

3A4

Buprenorphine is a _______ substrate

3A4

Topiramate is a ______ substrate

3A4

Sildenafil (Viagra, Revatio) is a _____ substrate

3A4

Flurazepam (Dalmane) is a ________ substrate

3A4

Particularly potent inhibitors of 3A4

clarithromycin
itraconazole
ketoconazle

Particularly potent inducer of 3A4

rifampin (90% reduction in serum concentrations of 3A4 substrates)

If you add rifampin to triazolam, what happens to the concentration of triazolam?

90% decreased concentration (ineffective)

What meds would lower the effectiveness of amphetamine

Dexamethasone
Rifampicin

What medications would lower the effectiveness of citalopram

Carbamazepine
Prednisone
Rifampicin

What medications would lower the effectiveness of bupropion

Rifampicin
Phenobarbital

What medications would lower the effectiveness of olanzepine?

insulin
Modafinil
phenobarbital
rifampicin

Drugs affected by tobacco

Theophylline
caffeine
tacrine
imipramine
haldol
estradiol

Cigarette smoking results in _______ clearing of heparin

faster (inducer)

Stopping cigarette smoking can result in caffeine __________

toxicity

Patients who smoke may have _____ levels of clozapine, haldol or olanzapine

lower serum concentration

The turnover time for CYP1A2 is __________

approximately 3 days with steady state reached at 1 week (important to remember when managing inpatients in smoke-free facility – can result win drug toxicity)

Cumin and turmeric ________ CYP1A2 activity

inhibit

blockage of voltage gated sodium channels
decreases glutamate release
potentiates GABA
weak inhibitor of carbonic anhydrase

topiramate

inhibition of alpha subunit of G protein associated with cAMP

Lithium

Indications:
acute mania

Lithium
valproic acid

Indications:
acute/mixed mania

Carbamazepine
Valproic acid
Risperidone
Asenapine

Indications:
Trigeminal neuralgia

Carbamazepine

Indications:
Partial seizures
Grand mal seizure patterns
Mixed seizure patterns

Carbamazepine

Indication:
complex partial seizures
simple and complex seizures
migraine prophylaxis

Valproic acid

What mood stabilizer is more likely to cause psoriasis and acne?

lithium

What mood stabilizer is more likely to cause thyroid dysfunction?

Lithium

What mood stabilizer has neuroprotective effects

Lithium

What mood stabilizer inhibits GSK-3 beta?

Lithium (plays role in tau phosphrylation)

What mood stabilizer increases bel-2?

Lithium (inhibits apoptosis)

What mood stabilizer is more likely to cause renal insufficiency?

Lithium

Indications:
maintenance and propylaxis of bipolar

Lithium
Risperidone

Indications:
mood stabilizer the augments antidepressant

Lithium

Indications:
schizoaffective (bipolar type)

Lithium

Indications:
Maintenance of bipolar II

Lamictal

Indications: Partial seizures in adult and children

oxcarbazepine

Indications: Lennox-Gaust syndrome seizures, partial seizures, tonic-clonic seizures, migraine prophylaxis

Topiramate

St John’s wort lowers the level of

Warfarin
Digoxin

Ginkgo Biloba contraindications

pt on antiplately or anticoagulant med

Garlic should be used cautiously with

anti platelet and anticlotting medications

After dc of benzodiazepine, rebound insomnia is likely to happen. What’s the next step in mgmt?

Start sedative antihistamine

What medications should be used with caution or avoided with ECT?

theophylline (can prolong seizure and status epileptics)
hypoglycemics
Beta blockers (can cause systole)
Lidocaine (reduce seizure induction)
Lithium (can prolong seizure and confusion)

What med if given with ECT can prolong seizure and confusion?

Lithium

What med if given with ECT can cause systole?

Beta blockers

What med if given with ECT can reduce seizure induction

Lidocaine

Alopecia is transient when taking valproic acid and can be treated with

a multivitamin containing zinc and selenium

Side effects of valproic acid are more frequent with ____________ than with other preparations

divalproex sodium

Carbamazepine and fluoxetine interaction

Increased carbamazepine levels

Carbamazepine and lithium interaction

increased risk of neurotoxicity

Carbamazepine and oral contraceptive (OBC) interaction

decreased levels of OBC

Carbamazepine and valproic acid interaction

increased carbamazepine levels
decreased valproic acid levels

Is phenytoin an inducer or inhibitor?

inducer

Is Valproic acid an inducer or inhibitor?

inhibitor

Is ciprofloxacin an inducer or inhibitor?

inhibitor

Is ketoconazole an inducer or inhibitor?

inhibitor

Indications: Schizophrenia

Quetiapine
Aripiprazole
Risperidone
Paliperidone
Olanzapine
Clozapine
Asenapine
Lurasidone
Ziprasidone
Lloperidone

Indications:
Acute psychosis

Ziprasidone

Indications:
agitation

Ziprasidone
Olanzapine
Aripiprazole

Indications:
schizoaffective

Clozapine
Paliperidone

Indications:
Autism (5-16 years)

Risperidone
Aripiprazole (all ages)

Antipsychotics FDA indicated for bipolar disorder

Ziprasidone
Olanzapine
Risperidone
Quetiapine
Aripiprazole

Which antipsychotic should you avoid in patients with a significant cardiac history?

(due to increased QTc)
Lloperidone
Ziprasidone

Patients tend to lose weight (no significant increase in lipids or glucose) when treated with this antipsychotic long term

Lurasidone

Antipsychotic that is category B for pregnancy, little prolactin increase

Lurasidone

What antipsychotic is related to mirtazapine and likely has antidepressant properties?

Asenapine

What antipsychotic has the notable side effect of metabolic syndrome and diabetic ketoacidosis?

Olanzapine

What antipsychotic has the notable side effect of dose-related hyperprolactinemia?

Risperidone

What antipsychotic has the notable side effects of metabolic syndrome and cataracts (in animal studies)

Quetiapine

What antipsychotic has no increase in prolactin?

Quetiapine

What antipsychotic has a black box warning of suicide in children

Aripiprazole

What antipsychotic is most likely to cause akathisia and can be activating at low doses

aripiprazole

What antipsychotic is preferred in Parkinson’s and Lewy body demential due to low EPS

Quetiapine

What antipsychotic has a very low risk of TD’s and no increase in prolactin?

Quetiapine

With what antipsychotic should you monitor with ophthalmological exam, AIMS, blood glucose, lipid profile, weight?

Quetiapine

With what antipsychotic should you monitor prolactin level?

Risperidone

With what antipsychotic should you monitor CBC?

Clozapine

What antipsychotic has potent alpha-1 blockade (prazosin like effects for PTSD related nightmares) and need to watch for orthostatic hypotension?

Lloperidone

What med causes notable side effects of agranulocytosis, seizures, myocarditis, cardiorespiratory failure

Clozapine

What antipsychotic has blood level available?

Clozapine

What antipsychotic can cause oral hypesthesia?

Asenapine

Major side effects of this antipsychotic include

Rare EPS
Prominent Sedation
Very low weight gain

What antipsychotic causes excess salivation?

Clozapine

What antipsychotic reduces suicide?

Clozapine

What antipsychotic can cause rebound psychosis and cholinergic rebound when abruptly withdrawing

Clozapine

What antipsychotic is preferred treatment of behavioral disturbance in children

Risperidone

There is a ________________ in patients taking anticonvulsants per black box warning

suicide and depression

Carbamazepine, phenytoin and oxcarbazepine are _______ of the CYP3A4 system and can ___________ levels of antidepressants and antipsychotics

inducer
lower

Barbituates and gapapentin have an increase risk of ________ in epileptic patients

depression

What is not a reasonable way to treat hypotension caused by an MAOI

introduce a small amount of cheese into diet (amount of tyramine differs among types of cheeses)

What are safe ways to increase BP in person taking MAOI

increase salt intake
fludrocortisone 0.6-0.8mg daily
wear support stocking
ensure adequate hydration
stimulant medication

When is the best time to measure plasma levels of lithium during therapy?

12 hours after last dose

A substance that decreases the metabolism of drugs

inhibitor

Is echinacea an inducer or inhibitor?

Inhibitor

Mechanism of atomoxetine

blocks reuptake of norepinephrine

Atomoxetin increases NE and DA levels by _____ in the _______

3x
prefrontal cortices
(no change in striatum or nucleus accumbans)

Increased NE has been shown to improve

depressive symptoms
fatigue
hypersomnia
motoric retardation
anhedonia

Common side effects of atomoxetine are

dry mouth
insomnia
constipation
urinary hesitancy

Valproic acid ________ the elimination half life of lamotrigine

doubles

Which med is used to tx alcohol use disorder by inhibiting a step in alcohol metabolism, leading to buildup of a toxic metabolite

Disulfiram

Disulfiram blocks _______ leading to

aldehyde dehydrogenase
unpleasant hangover like side effects from drinking ETOH (accumulation of acetaldehyde)

______ and ______ block the activation of the reward system associated with alcohol and opiates

Naltrexone
Nalmefene

Cognitive affects of taking benzodiazepines are greatest in _____

elderly (especially at risk is pts with delirium and dementia)

Initial side effects of benzodiazepines can persist and worsen in pts

with liver disease
taking longer-acting agents (diazepam)

What antidepressants significantly increase dopamine?

Phenelzine
Selegiline
(Bupropion is slight)

Which antidepressants significantly increase NE?

Desipramine
Amitriptyline
Buproprion
Duloxetine
Phenelzine
Selegiline

Which antidepressants increase does not increase DA or 5-HT but increases NE?

Desipramine

Which antidepressants increase NE and 5-HT, but not DA

Amitriptyline
Duloxetine
Effexor

Which antidepressants increase NE, 5HT and DA

Phenelzine
Selegiline

_____ of every _____ black or white persons may have an exaggerated response to beta-blockers due to a polymorphism in CYP450 enzymes

1
15

Sialorrhea

excessive salivation

Sialorrhea is associated with side effects of

Clozapine (31-48%)

Clozapine has a boxed warning concerning

AGRANULOCYTOSIS (severe neutropenia is associated with significant risk of serious infection or death)

orthostatic hypotension
bradycardia
syncope
(more likely to occur during initiation of tx and titration of dose)

Baseline ANC must be at least ___________ before initiation of Clozapine tx

1500

Due to _________ clozapine is available only under a Risk Evaluation Mitigation Strategy (REMS) program.

risk of severe neutropenia (ANC<500)

Acute alcohol ingestion _______ the rate of drug metabolism

lowers (inhibits)

Acute alcohol ingestion causes benzodiazepine level to

increase

Hypnotic drug of choice for pt with insomnia who wants to take it in the middle of the night without causing significant grogginess in the morning

Zalepon

Half life of Zalepon

1-2 hours

What is the mechanism of action of galantamine?

competitive inhibitor of acetylcholinesterase

According to the Lilly Worldwide Pharmacovigilance Safety Database, what is the rate of spontaneous abortion in olanzapine exposed pregnancy?

13% (falls within range of normal historic control rates)

Exposure during gestation to this medication can cause arrhythmias, polyhydramnios, nephrogenic diabetes insidious, and “floppy infant syndrome”

LIthium

Desvenlafaxine is ________ protein bound with ____ likelihood of displacing tightly protein-bound medication like phenytoin and warfarin

weakly
reduced

The SNRIs have an ______ onset of action compared to SSRIs due to _________

earlier
rapid down-regulation of beta-adrenergic receptor-coupled cAMP

Do stimulants cause an increase in tics

no (can worsen for a short interval, then go back to baseline)

Most prescription and nonprescription medications can be used safely with _________

stimulants (MAOI formally contraindicated but can be used with caution in tx resistant cases)

Excessive caffeine intake with stimulants may _________ the effectiveness of the stimulant

decrease (increase sleep problems)

Use of stimulants with anticonvulsants may require

dosage adjustment and blood levels of both medication due to decreased effects of anticonvulsants

SSRI associated with the greatest weight gain

Paroxetine

Amiloride is used to treat which side effect of lithium

Renal side effects

TCA for OCD

Clomipramine

Induction of a seizure with ECT occurs when an applied electrical stimulus causes which of the following?

Depolarization of cell membranes of neurons in brain synchronously

A 14 year old boy is dced on carbamazepine after an admission for acute mania. About 4 weeks later, he is again acutely manic despite having been initially stabilized on carbamazepine. What is the likely cause of this medication failure?

Autoinduction

transcription up regulation of genes involved in its own metabolism

Autoinduction

How long does auto induction of carbamazepine take

3-5 weeks (after ignition of a fixed dose when taken consistently)

Meds that can increase serum concentration of 3A4 temporarily (but ultimately act as an inhibitor)

Cimetidine
Erythromycin
Grapefruit juice

Vagus nerve stimulation is FDA approved for

epilepsy
treatment resistant depression

it involves surgical implantation of a device in the carotid sheath

vagus nerve stimulation

Adding ginseng to lithium can cause

irritability, insomnia and mania

Adding ginseng to neuroleptics or phenelzine can cause

irritability, insomnia and mania

What adverse reaction is associated with mirtazapine use?

Agranulocytosis (1/1000)

Mirtazapine ______ nausea and vomiting due to ________

improves
5-HT3 receptor blockade

Urinary ______ can occur with mirtazapine

frequency

Kava acts on ________ receptors and long term use has been linked with ___________

GABA
liver toxicity

Foods that act as inducer

Cauliflower
leafy green vegetables
broccoli
carrots
licorice

Foods that act as inhibitor

grape (resveratrol)
grapefruit juice
cranberry
pomegranate
mango
red peppers
green tea
vitamin E
kava kava root

Cocaine during pregnancy has been associated with

Uterine Growth Restriction (IUGR)
Placental Abruption
Small for gestational age (SGA)

Carbamazepine is ______ for mothers who are breastfeeding

Safe

Valproid acid is ______ for mothers who are breastfeeding

safe

Phenytoin is ______ for mothers who are breastfeeding

safe

Phenobarbital is ______ for mothers who are breastfeeding

safe

Primidone is ______ for mothers who are breastfeeding

safe

Lithium is ______ for mothers who are breastfeeding

unsafe (EKG changes and toxicity)

Lamotrigine is ______ for mothers who are breastfeeding

unsafe (risk of fatal rash)

Clonazepam is ______ for mothers who are breastfeeding

unsafe (CNS/respiratory depression)

Can cause hyponatremia especially when given with SSRI

Oxcarbazepine

Lithium _______ affect the efficacy of oral contraceptives at any dose

does not

Carbamazepine_______ affect the efficacy of oral contraceptives at any dose

does (decreases efficacy)

Another name for Carbamazepine

Tegretol

Topiramate _______ affect the efficacy of oral contraceptives at any dose

at doses >200mg does (decreases efficacy)

Oxcarbazepine _______ affect the efficacy of oral contraceptives at any dose

does (decreases efficacy)

A 64 year old man with early stage Parkinson’s disease takes the MAO-B inhibitor selegiline for symptomatic relief. Why are MAO-B inhibitors favored over MAO-A inhibitors for treating Parkinson’s disease

MAO-A normally metabolized tyramine

Two MAO-Bi drugs that have been approved by FDA without dietary restrictions for early stage Parkinson’s (but lose selectivity at high-dose)

Selegiline
rasagiline

MAO-B inhibitors principally block

dopamine
phenylethylamine

What will not increase lithium levels?

High sodium diet

A low sodium diet is likely to ______ lithium

increase

Toxicity of Lithium occurs at levels marked by tremor, GI symptoms, blurred vision, increased DTR, vertigo, and confusion

>1.5 meq/L

Life-threatening toxicity of Lithium occurs at

>2.0 meq/L

At Lithium level of ___________ seizures, coma, arrhythmias, and permanent neurological impairment may occur

>2.5 meq/L

In acute toxicities with lithium levels >4.0 meq/L ________ is the only way to get lithium out of the bloodstream

hemodialysis

Medications associated with higher level of Lithium

diuretics
ACE
NSAIDs
COX-2 inhibitors

Lifestyle risk factors for elevated Lithium level

Salt-restricting diets
dehydration
high heat
poor PO intake

Lithium levels should be checked _____ after an increase or any time prior to an increase

5 days (sooner in acute mania)

Target level for Lithium in tx of mania

0.8-1.0 meq/L

Fewer lithium side effects (but more manic episodes) are seen in this level

0.4-0.6meq/L

In stable patients on chronic lithium treatment, levels should be drawn every ______ and renal/thryoid function every ______

6 months
6-12 months

When taking lithium, renal function should be evaluated every ______ months during the first 6 months of treatment

2-3 months

Sexual side effects from antipsychotics are due to

alpha-2 receptors

Long term effects of typical antipsychotics

weight gain
photosensitivity
sexual side effects
hyperprolactinemia

TD is associated with all _______ and occurs at a rate of approximately _______ per year

typical antipsychotics
5%

Whta is the most effective treatment for major depression?

ECT

Valproate competes with lamotrigine at _______________

glucuronidation site 2B7

Anything that blocks either _______ or ________ will increase lamotrigine levels

(UGT) 1A4 or 2B7

Depakote competes with lamotrigine at _______________

glucuronidation site 2B7

Tamoxifen (Novadex) competes with lamotrigine at _______________

glucuronidation site 1A4

Imipramine competes with lamotrigine at _______________

glucuronidation site 1A4

Amitriptyline (Elavil) competes with lamotrigine at _______________

glucuronidation site 1A4

Doxepin competes with lamotrigine at _______________

glucuronidation site 1A4

Nicotine competes with lamotrigine at _______________

glucuronidation site 1A4

Olanzapine competes with lamotrigine at _______________

glucuronidation site 1A4

Promethazine (Phenergen) competes with lamotrigine at _______________

glucuronidation site 1A4

Meperidine (Demerol) competes with lamotrigine at _______________

glucuronidation site 1A4

Clozapine (Clozaril) competes with lamotrigine at _______________

glucuronidation site 1A4

Clorpromazine (Thorazine) competes with lamotrigine at _______________

glucuronidation site 1A4

Androsterone and Testosterone competes with lamotrigine at _______________

glucuronidation site 1A4

Progestins (progesterone) competes with lamotrigine at _______________

glucuronidation site 1A4

Diphenhydramine (Benadryl) competes with lamotrigine at _______________

glucuronidation site 1A4

Asenapine (Saphris) competes with lamotrigine at _______________

glucuronidation site 1A4

Atorvastatin (Lipitor) competes with lamotrigine at _______________

glucuronidation site 2B7

buprenorphine/naloxone competes with lamotrigine at _______________

glucuronidation site 2B7

Ibuprofen (Advil, Motrin) competes with lamotrigine at _______________

glucuronidation site 2B7

Fenofibrate (Tricor) competes with lamotrigine at _______________

glucuronidation site 2B7

Oxycodone/APAP (Percocet) competes with lamotrigine at _______________

glucuronidation site 2B7

Hydromorphone (Dilaudid) competes with lamotrigine at _______________

glucuronidation site 2B7

Methadone competes with lamotrigine at _______________

glucuronidation site (2B7)

Tacrolimus (Prograf) competes with lamotrigine at _______________

glucuronidation site 2B7

Codeine competes with lamotrigine at _______________

glucuronidation site 2B7

Temazepam (Restoril) competes with lamotrigine at _______________

glucuronidation site 2B7

Valproic Acid (Depakote) competes with lamotrigine at _______________

glucuronidation site 2B7

Simvastatin (Zocor) competes with lamotrigine at _______________

glucuronidation site 2B7

carvediolol (Coreg) competes with lamotrigine at _______________

glucuronidation site 2B7

Cyclosporine competes with lamotrigine at _______________

glucuronidation site 2B7

Lorazepam (Ativan) competes with lamotrigine at _______________

glucuronidation site 2B7

Oxazepam (Serax) competes with lamotrigine at _______________

glucuronidation site 2B7

Benzodiazepines that undergo glucuronide conjugation 2B7 (not via the cytochrome p450 system)

Oxazepam
Temazepam
Lorazepam
(OTL mnemonic: Outside The Liver).

What drug has been associated with insulin-like effect responsible for lowering blood glucose and increasing appetite and weight gain?

Lithium

What drug has been associated with myocarditis?

Clozapine

What drug has been associated with eosinophilic colitis

Clozapine

In studies, which ECT modality reduces cognitive side effects but is less effective in terms of symptom reduction?

unilateral non dominant

A patient discharged on valproate has been stable for many years on lamotrigine, and you would like to initiate therapy with this medication and reach a therapeutic level before tapering off the valproate. What would be the starting dose of lamotrigine?

Half of what it would normally be since half life effect is doubled

Between olanzapine, clozapine, quetiapine, and haldol, which is least associated with sedation?

Haldol

3 antipsychotics with high anticholinergic effects

clozapine
thioridazine
chlorpromazine

4 antipsychotics associated with high sedation

clozapine
quetiapine
thioridazine
chlorpromazine

Olanzapine is associated with _______ sedation

mild

What antipsychotic is not associated with EPS?

Clozapine

Risperidone is _____ anticholinergic, ______ sedation _______ EPS

low
low
low

Haloperidol is ______ anticholinergic, _______ sedation, ______ EPS

low
low
high

Thiroidazine is _____ anticholinergic, _____ sedation, ____ EPS

high
high
low

Chorpromazine (Thorazine) is ______ anticholinergic, ______ sedation )______ EPS

high
high
low

Quetiapine is _____ anticholinergic, ______ sedation, ______ EPS

low
high
low

Olanzapine (Zyprexa) is ______ anticholinergic, ______ sedation, ______ EPS

low
mid
low

Ziprasidone (Geodon) is ______ anticholinergic, _____ sedation, _____ EPS

low
low
low

Aripiprazole (Abilify) is _______ anticholinergic, ______ sedation, ______EPS

low
low
low

Another name for Abilify

Aripiprazole

Another name for Aripiprazole

Abilify

Another name for Clozapine

Clozaril

Another name for Clozaril

Clozapine

Another name for Seroquel

Quetiapine

Another name for Quetiapine

Seroquel

Another name for Invega

Paliperidone

Another name for Paliperidone

Invega

Another name for Thioridazine

Mellaril

Another name for Mellaril

Thioridazine

Another name for Clorpromazine

Thorazine

Another name for Thorazine

Clorpromazine

Another name for Tegretol

Carbamazepine

Another name for Valproate

Depakote

Another name for Depakote

Valproate Sodium
Valproic Acid
Divalproex Sodium

Another name for Valproic Acid

Valproate Sodium
Depakote
Divalproex Sodium

Another name for Oxcarbazepine

Trileptal

Another name for Trileptal

Oxcarbazepine

Another name for Luvox

Fluvoxamine

Another name for Fluvoxamine

Luvox

Another name for Prozac

Fluoxetine

Another name for Fluoxetine

Prozac

Another name for Sertaline

Zoloft

Another name for Zoloft

Sertraline

Another name for Paroxetine

Paxil

Another name for Paxil

Paroxetine

Another name for Lexapro

Escitalopram

Another name for Citalopram

Celexa

Another name for Celexa

Citalopram

Another name for Escitalopram

Lexapro

Another name for Cymbalta

Duloxetine

Another name for Duloxetine

Cymbalta

Another name for Effexor

Venlafaxine

Another name for Venlafaxine

Effexor

Another name for Pristiq

Desvenlafaxine

Another name for Desvenlafaxine

Pristiq

Pt’s meds include lisinopril, atorvastatin, bupropion, ibuprofen. Which medication is responsible for his insomnia?

Bupropion

Bupropion is also known as

Wellbutrin

Wellbutrin is also known as

Bupropion

What SSRI is known to cause activation and insomnia

Fluoxetine

There are no _________ for ECT

ABSOLUTE contraindications

Space-occupying intercerebrayl lesion has traditionally been considered a ______

contraindication of ECT since ECT increases intracerebral pressure, however slow growing meningiomas without mass effect and other similar lesions do not present a high risk

While caution needs to be applied weighing risks and benefits, ECT has been safely administered (after appropriate subspecialty consultation) to pts with a hx of

brain surgery
deep brain stimulators
cardiac pacemakers
AICD
vagus nerve stimulator
seizures
space occupying intracerebral lesions

ADHD patients may have side effects from stimulants that represent

manifestation or exacerbation of comorbid disorder (ie GAD)
Side effect of treatment

Antihistamine _______ is used to increase appetite

Cyproheptadine

Medications to treat sleep problems associated with stimulant use

Melatonin
Clonidine
Diphenhydramine
Trazodone
Mirtazapine

Hyperparathyroidism is associated with which medication

Lithium

hyperparathyroidism

excessive levels of parathyroid hormone

Ketoconazole is ______ of the ______ enzyme

inhibitor
3A4

Cannabinoids is a _______ of the ______ enzyme

inhibitor
3A4

Phenytoin, Carbamazepine, and St John’s wort have this in common

3A4 inducer

Ketoconazole, amiodarone, and erythromycin have this in common

3A4 inhibitor

Regarding ECT treatment for depression in adolescents, which of the following statements is correct

response to ECT is similar in adolescents and adults

Before ECT is used on an adolescent

2 child and adolescent psychiatrists who are not the primary caregivers for the adolescent should be consulted and agree that it is indicated

Which antidepressant is least likely to cause significant drug-drug interactions in the elderly

Venlafaxine (most SSRIs cause some inhibition of the 2D6 enzyme which leads to increased plasma level of other drugs (ie antiarrhythmics, benztropine, TCA, many antipsychotics)

A 26 year old F with dx of bipolar on lithium presents to ED with marked tremor, ataxia, and confusion after a suspected suicide attempt less than an hour before admission. Her lithium level is 2.1. Over the next half hour, the pt becomes increasingly confused and then unresponsive, what is the next step in management?

Alert nephrology for emergent hemodialysis

Which medication has been helpful in women with stress incontinence

Duloxetine

Another name for Lurasidone

Latuda

Another name for Latuda

Lurasidone

Indicated for bipolar depression

Latuda (Lurasidone)

What medication given in combination with Latuda may cause orthostatic hypotension

beta blocker

Which substance or technique was first used to induce convulsion to treat psychiatric illness and catatonia

Camphor

What medication when given with SSRI is most likely to predispose patients to serotonin syndrome

Meperidine (Demerol)

Valproic acid is superior to lithium in patients with

mixed symptoms
prominent depressive symptoms during mania
multiple prior mood episodes

________ is more effective than Valproic acid in reducing symptoms of acute mania

Olanzapine

Valproic acid and _________ are equally efficacious in the treatment of acute mania

haldol (response rate 48-53%)

Divalproex sodium is also known as

Depakote
Valproic acid
Valproate Sodium

Max dose of Divalproex Sodium

60mg/kg/day

Depakote is also known as

Divalproex Sodium
Valproic Acid
Volproate Sodium

Carbamazepine may ______ the effectiveness of Risperidone

Decrease

Carbamazepine is metabolized in the ____ by ____ substrate

liver
3A4

________ requires monitoring for signs of bruising and bleeding

Carbamazepine

What medication causes metabolic acidosis via carbonic anhydrase inhibition?

Topiramate

What does metabolic acidosis via carbonic anhydrase inhibition initiate?

kidney stone formation

Carbamazepine acts as a CYP 450 3A4 ____, ____ the effectiveness of other CYP 3A4 substrates

inducer
lowering

What anti-epileptic medication causes bone marrow suppression, resulting in agranulocytosis and leukopenia

Carbamazepine (Tegretol)

What antidepressant should be used in caution with a patient taking a protease inhibitor (ritonavir, saquinavir) because of toxic interaction

Nefazodone

Nefazodone is also know as

Serzone

Serzone is also known as

Nefazodone

Trazodone is also known as

Desyrel

Desyrel is also known as

Trazodone

Nefazodone and Trazodone are both ________ blockers

5-HT2a

Nefazodone and Trazodone side effects

sedation, GI distress, postural hypotension (alpha block), trazaBONE-sexual side effects

Which drugs is only an antiepileptic (not used as a mood stabilizer)?

Levetiracetam

What is the minimum amount of time to continue maintenance ECT treatment to achieve the best outcome after pt has discharged. He responded to 3 administrations of ECT?

6 months

3 specific indications for continuation ECT are

1. hx of recurrent episodes responsive to ECT
AND

2. ineffectiveness of or intolerance to prophylactic pharmacotherapy
OR
3. patient preference

Average duration of continuation ECT

10 weeks

Continuation ECT following successful ECT typically involves gradual shifting from

frequent weekly to monthly treatments over 1-3 months and then maintaining the monthly administration schedule for at least 6 months after remission

Long term use of continuation maintenance ECT is indicated when

pt hx suggest high risk of delayed relapse on medication alone
or
evidence of decompensation occurs during attempts to stretch interval between treatments during the continuation phases

When someone is on continuation maintenance ECT, the need should be reviewed by practitioner and patient at least

twice a year

Consent for continuation maintenance ECT should be reobtained at least every

6 months

What needs to be reevaluated every 6 months with continuation maintenance ECT

need/indication
anesthetic/medical complications
cognitive tests (presence of persistent memory deficits weighed against anticipated benefits)

Even in patients who do not have a hx of diabetes, a possible side effect of olanzapine

Diabetic ketoacidosis
Hyperosmolar nonketotic coma

What is considered a side effect of antipsychotics and is treated by beta blockers (propranolol)

Akathisia

Nefazodone is a potent ________ of CYP 450

inhibitor

Medication can be used as an antidote to SSRI induced impotence

Nefazodone

Unlike other antidepressants, this medication enhances rapid eye movement (REM) and may increase restful sleep in some patients

Nefazodone

This antidepressant medication is not usually given first line for depression or at all anymore due to risk of liver failure (need to do liver function tests)

Nefazodone

Haldol should be avoided in

pregnant women (d/t risk for birth defects)

Common reasons for inadequate response to antidepressants include

inadequate dosing
pt non-compliance
misdiagnosis
substance abuse (especially ETOH)
Dc medications too quickly/early

Young female brought to ED after being found in coma next to empty bottle of Vicodin. She is unresponsive. Medication that is indicated in management

Naloxone

Naloxone is also called

Narcan

Narcan is also called

Nalaxone

Flumazenil is also called

Romazicon

Naltrexone is also called

ReVia

Which MAOI antidepressant has the lowest incidence of sexual dysfunction?

Tranylcypromine

Which MAOIs have a high incidence of sexual dysfunction

Isocarboxazid
Phenelzine

Tranylcypromine is also known as

Parnate (MAOI)

Isocarboxazid is also known as

Marplan (MAOI)

Phenelzine is also known as

Nardil (MAOI)

The 2015 Beers criteria contains a strong recommendation against using SSRI in combination with other __________ in older adults based on moderate evidence for __________

CNS acting medications
increased fall and fx risk

Memantine is also known as

Namenda

Memantine mechanism of action

NMDA receptor antagonist

Potent allosteric potentiating ligand of human nicotinic acetylcholine receptors

Galantamine

Galantamine is also known as

Razadyne

an acetylcholinesterase inhibitor that inhibits both butyrlcholinesterase and acetylcholinesterase

Rivastigmine

Selectively inhibits acetylcholinesterase unlike Rivastigmine

Donepezil

Donepezil is also known as

Aricept

Precursor to the neurotransmitters dopamine, norepinephrine and epinephrine (collectively known as catecholamines)

L-DOPA

2 typical antiparkinsonian drugs that are ineffective in the tx of antipsychotic-induced Parkinsonism

Levodopa
Selegiline

antipsychotic-induced Parkinsonism medication that works for tx, but its effects tend to wear off

Amantadine

Lithium is associated with risk of Ebstein’s anomaly which is malformation of which structure

Tricuspid valve

What is true regarding methylphenidate and amphetamine?

Both stimulants increase intrasynaptic levels of NE and Dopamine

Through which receptor dose mirtazapine exert its effect

Alpha-2 adrenergic antagonism.

What do oral contraceptives, reserpine, clonidine, ETOH, hydralazine, amantadine, benzodiazepines, and guanethidine have in common

can cause depression

Reserpine (Serpasil)

peripherally-acting sympathetic antagonist used for HTN; depletes stores of catecholamines in neurons; decreases TPR and CO

Guanethidine (Ismelin)

adrenergic neuron blocker
can cause depression

Loxapine (Loxitane)

typical tetracyclic antipsychotic with antidepressant properties

It’s active metabolite is amoxapine which is a secondary amine tricyclic antidepressant

Loxapine (Loxitane)

Patients with atypical depression respond particularly well to this MAOI

Phenelzine (Nardil)

What is the most appropriate medication for melancholic depression in the geriatric population?

Nortriptyline

Nortriptyline is also called

Pamelor

Which of the following is the safest short acting barbiturate that can be used as an induction for anesthesia?

Sodium thiopental

We have an expert-written solution to this problem!

What medication should be avoided in patient with hx of epilepsy and hyperlipidemia who wants to quit tobacco use?

Bupropion

What is the best medication option for pt who wants to quit tobacco use who has a hx or seizures

Varenicline
can be combined with NRT

Nephrolithiasis occurs in about _______ of topiramate pts

15%

Associated with acute myopia and secondary closure glaucoma, causing ocular pain and blurred vision d/t increased intraocular pressure

Topiramate

Metabolic acidosis is associated with ______, _____ levels should be monitored during use

topiramate
serum bicarbonate

These side effects do not improve over time with topiramate use

Decreased appetite
paresthesias
cognitive slowing

Weight loss is a significant side effect of

topiramax

Placing electromagnetic coil on scalp and running electricity through it

TMS

When was ECT first used in the US

1940s

Which TCA has been found in considerably higher concentration in breast milk compared to other medications in the same class?

Doxepin

Doxepin is also known as

Sinequan (TCA)

Inhibits voltage-dependent sodium channels and presynaptic sodium channels

Carbamazepine (Tegretol)

Increases GABA in brain

Depakote

Inhibits glutamate releases

Carbamazepine

Inhibits sodium channels, enhances GABA-a receptors, and antagonizes kainite at AMPA receptors

Topiramate

Duloxetine should not be given to patients who

consume large amounts of alcohol or have chronic liver disease

Duloxetine has a ________ half-life than venlafaxine

Longer (dc syndrome less likely)

Duloxetine increases this enzyme

hepatic transaminases

What do charcoal broiled beef, tobacco, marijuana, modafinil, omeprazole and cruciferous vegetables have in common?

1A2 inducer

What do caffeine, clozapine, duloxetine, imipramine, olanzapine, ramelteon and amitriptyline have in common?

1A2 substrate

What do fluvoxamine, cimetidine, ciprofloxacin, grapefruit juice, ketoconazole have in common

1A2 inhibitors

Which pair is least likely to cause weight gain?

Lamotrigine and topiramate

Which pair of antipsychotics is most likely to cause weight gain

Clozapine and olanzapine

Which pair of mood stabilizers is most likely to cause weight gain?

Valproic acid and lithium

Drugs that can induce mania

stimulants
bromides
cocaine
antidepressants
isoniazid
procarbazine
steroid
amphetamines

Medical conditions that can cause mania

syphilis
brain tumors
influenza
Q fever
multiple sclerosis
delirium

Which of the following symptoms characterizes the toxicity of clomipramine

Convulsions and coma

Mechanism of action of Clomipramine

blocks reuptake of NE and 5-HT (TCA)

Patients require ______ weekly for the first ______ months if they are using clozapine to monitor ________

CBC with a differential
6
ANC for agranulocytosis

When starting clozapine, CMP, Fasting lipid and fasting glucose should be monitored at some point within the first ____ of treatment to screen for metabolic syndrome, then if check does not demonstrate sx of metabolic syndrome, screening can be done ______

3 months
annually

Which medication side effect is NOT associated with lithium or valproic acid

eosinophilic colitis

Pancreatitis is a rare but potentially deadly side effect of

valproic acid

Hepatic failure is associated with

valproic acid

hyperparathyroidism and hyperglycemia have been associated with

Lithium

Hyperplasia and ademomas of parathyroid glands have been associated with

Lithium

Mechanism of action of varenicline

partial agonist of alpha4beta2 and full agonist of alpha7

mechanism of action of bupropion (Zyban) that helps with smoking cessation

partial agonist of alpha4beta2 and partial agonist of alpha7

mechanism of action of NRT that helps with smoking cessation

Full agonist at alpha4beta2 and full agonist at alpha7

Neuroleptics and antiemetic drugs such as metoclopramide are dopamine _____

receptor antagonists

Rifampicin is an __________ of CYP _______

inducer
2D6

Clozapine leads to seizures in ____ of patients taking >=6–mg

4%

___________ is most likely to cause seizures among typical antipsychotics

Chlorpromazine

Which drug is effective in improving sexual dysfunction in men but when given with an antidepressant to counter its adverse sexual effects can be counterproductive because it is actually anxiogenic?

Yohimbine

Can help reverse some sexual dysfunction caused by SSRIs at 4-12mg/day, but also can reverses the antidepressant or anti-obsessive effects of SSRIs

Cyproheptadine

The use of _____ has been more effective than placebo in men with SSRI-induced sexual dysfunction

Sildenafil (Viagara)

______ Has been reported to increase overall sexual satisfaction in both men and women

Sildenafil (Viagara)

Valproic acid is a CYP450 ________

inhibitor

What is NOT true regarding typical antipsychotic overdose

Dialysis is useful in treating it. (It is often lethal due to significant cardiovascular effects)

What seizure medications are considered safe for use in breastfeeding mothers

Carbamazepine (Tegretol)
Valproic Acid (Depakote)
Phenytoin (Dilantin)
Phenobarbital
Primidone (Mysoline)

If >______ %of the body surface area is involved it is termed Toxic Epidermal Necrolysis

>30

In it involves _______% of the skin surface area it is known as Steven-Johnson Syndrome (SJS)

<10

If it involves ______% of the skin surface area it is known as SJS-TEN

10-30

Mr Matthews presents for evaluation of this out-of-control aggressive impulses. Usually he is “a regular guy with decent temperament” but sometimes becomes disproportionately aggressive to a precipitating stress. Which of the following medications may worsen the symptoms of intermittent explosive disorder long term?

Diazepam

_________ and ______ have been superior to placebo in reducing impulsive aggression in pts with intermittent explosive disorder

Fluoxetine
Oxcarbamazepine

______ might help control explosive rage

Beta blockers

Due to risk of neural tube defects in bipolar pts on lithium, carbamazepine or valproic acid maintenance, ___________ must be monitored, if elevated, ______ and _______ are needed to look for _______

alpha-fetoprotein
amniocentesis
ultrasound
spinal deformities

Due to the risk for cardiac abnormalities in pts on lithium, valproic acid, or carbamazepine maintenance during pregnancy, _______ should be suggested at ________ weeks to rule out cardiac effects

high-resolution ultrasound
16-18

As a result of large fluid shifts during delivery, the bipolar medication levels of ______________ can become toxic so dose should be ______ and hydration should be ______. Discontinuation is not recommended due to _____

lithium
carbamazepine
valproate
lowered
adequate
50% risk of postpartum relapse

Buspirone mechanism of action

5HT1A receptor partial agonist at postsynaptic receptors
acts on postsynaptic receptors in hippocampus and cortex as partial agonist
Mild-moderate presynaptic dopamine agonist at D2, D3, and D4 receptors
Partial alpha-1 receptor agonist

The DSM-5 now includes _____ separate criteria for intermittent explosive disorder _____ which have empirical support

2
A1 – does not lead to destruction of property or physical injury
A2- 3 outburst that involve injury or destruction within a year period

In order to be diagnosed with intermittent explosive disorder, individual must be at least ____ old

6 years

What is true about Carbamazepine

It slows cardiac conduction and should be avoided in pts with high-grade AV block or sick sinus syndrome

SSRI discontinuation syndrome symptoms usually resolve within

2 weeks

Common side effects of stimulant medications

Abdominal pain
anorexia
diminished appetite
headache
insomnia
mood lability
nausea
nervousness
vomiting

What SNRI is associated with discontinuation syndrome

Venlafaxine

Concurrent administration with which medication is likely to decrease levels of valproic acid?

Carbamazepine
Phenytoin
Primidone
Phenobarbital

Drugs that may increase valproic acid levels

cimetidine
macrolide antibiotics (erythromycin)
felbamate

Which SSRI would cause the most problems in a pt on Clozaril

Fluvoxamine

FLuovoxamine can increase sedation effects caused by

alprazolam
triazolam
trazodone

For a patient with a GFR of 23, what is a good mood stabilizer that is hepatically excreted?

Valproate

When assessing cognitive elements in a MSE, which of the following elements is assessed by using proverbs such as “The golden hammer breaks the iron door.”

Abstraction

Of the >50 different CYP450 enzymes _____ enzymes metabolize approximately ______ of all medications

6
90%

Enzyme inhibition is a ______ process than enzyme induction

faster

Enzyme induction generally occurs over the course of ______

1-2 weeks

Who exhibits enzyme inhibition

premature infants
elderly men and women
patients with liver disease

What medication is a tetracyclic antidepressant

mirtazapine

OD of this medication presents as dizziness, ataxia, ophthalmoplegia, hyponatremia, and thrombocytopenia

Carbamazepine

Drugs tend to be absorbed more slowly when a patient has a full stomach. Which medication should be taken on an empty stomach?

Sedatives/hypnotics

What is the difference in side effects between SNRIs and SSRIs

SNRIs may cause HTN
SSRIs cause more wt gain, especially paroxetine
SNRIs cause less sexual dysfunction when compared to SSRIs
Withdrawal tends to be worse with SNRIs

Which of the following is a three question screening tool used for dx of ETOH abuse and dependence in pt who is acutely ill or otherwise poorly disposed to speak with the practitioner?

AUDIT-C

AUDIT has ____ multiple choice questions regarding the _________ and is the ______ for screening for identification of _______

10
quantity and frequency of alcohol consumption, drinking behavior and alcohol related problems
Gold standard
Alcohol Use Disorders

CAGE is a _____ item screening took with a ______ sensitivity and ______ specificity for AUD

4
50-75%
80%

C in CAGE

Have you ever felt you should Cut down on your drinking?

A in CAGE

Have people ANNOYED you by criticizing your drinking?

G in CAGE

Have you ever felt bad or guilty about your drinking?

E in CAGE

Have you ever had a morning eye-opener

MAST stands for

Michigan Alcohol Screening Test

MAST has ____ screening questions with ______ answers

25
yes/no

On administration of a drug that causes CYP2D6 inhibition, which side effect of methylphenidate will increase?

none because it does not have any significant metabolism via that enzyme and the majority of metabolism occurs at first pass and outside the liver

Due to a high risk for dependence, for which hypnotic must a test dose be given if a pt’s dose is not known

Barbiturates

If a pt is given too large of a dose of Barbituates, this will happen

nystagmus
slurred speech
ataxia
sedation

What change in the dose-response curve is observed when a pt becomes tolerant of a medication

shift to right

Buspirone primarily acts on which receptor

5-HT1A

Which medications (not yet FDA approved) can be used to treat kleptomania?

Fluoxetine
Naltrexone

Which of the following is characteristic of fetal phenytoin syndrome

Hypertelorism

Hypertelorism

eyes spaced widely apart

fetal valproate syndrome features

bifrontal narrowing
midface hypoplasia
broad nasal bridge
short nose with anteverted nares
epicanthic folds
micrognathia
shallow philtrum
thin upper lip
thick lower lip

NSAID ______ Lithium clearance and can cause side effects such as ______

decrease
tremor

What antidepressant is associated with significant sexual side effects and weight gain

Phenelzine

Some patients on chronic lithium treatment may have gradually _______

increased creatinine

Development of hypothyroidism that is not responsive to thyroxine constitutes a valid reason to consider

discontinuation of lithium

ECG effects common in LIthium

changes in depolarization phase
worsening existing arrhythmias

With most modern devices, the dosage of ECT is changed by varying which aspect

time of exposure to fixed current

When drug is quickly metabolized to inactive metabolites, which of the following statements may be true of this patient

She is a poor metabolizer

Donazepil is believed to improve

memory in neurocognitive disorder by increasing the amount of acetylcholine available in the synaptic cleft by inhibiting its breakdown by acetylcholinesterase. This inhibition is reversible.

Barbituates ________ plasma TCA, _____ the antidepressant effects because of their induction of __________ and accelerated _______

lower
diminishes
liver microsomal enzymes
degradation of the TCA

Barbiturates act on ______ receptors

GABA

For gastrointestinal issues associated with Lithium use

change to longer-acting oral preparation or use lithium citrate syrup

For lithium associated edema

administer spironolactone (25mg/day) and follow lithium levels closely

For lithium tremor

administer propranolol (10-30mg TID) or primidone (25-100mg/day) as second-line

For lithium hypothyroidism treat with

thyroid hormone and continue lithium therapy

When in the body is melatonin made

Pineal gland (Serotonin derivative)

Hormones hypothalamus makes

TRH (thyrotropin releasing hormone)
dopamine
GHRH (growth hormone releasing hormone)
SS (somatostatin)
GnRH (gonadotropin releasing hormone)
CRH (corticotropin releasing hormone)
oxytocin
vasopressin (ADH)

The posterior pituitary stores and releases

oxytocin and vasopressin (ADH)

The anterior pituitary makes

ACTH
TSH
FSH
LH
GH
prolactin

What pharmacological treatment is clinically supported for treating kleptomania

Lithium

Which of the following medication has the least effect on the CYP-450 system: carbamazepine, valproate, levetiracetam, phenytoin, Lamictal

Levetiracetam

Which of the following hypnotic agents is preferred for the elderly

Ramelteon

What is the rate of malformation among fetuses exposed to SSRI medications

2.6% (consistent with reports in general population)

Studies suggest that TMS may be most useful for which symptom in schizophrenia?

Hallucinations

Second generation drugs are more effective for treating ________ in schizophernia

negative symptoms

Used for aggression and impulsivity in schizophrenics

Beta blockers
Lithium

ECT is most often used for ______ in schizophrenia

catatonia

TMS is effective for _____ in OCD

impulsivity

Second generation antipsychotic with high D2 blocking/binding

Risperidol

Second generation antipsychotic with lower D2 blocking/binding

Clozapine
Quetiapine

Blockade of ______ or more of the D2 receptors is associated with increased EPS

80%

Lower D2 blockade of ______ % is safer and still clinically effective

60-70%

Bupropion is an NDRI that should be avoided in patients with

anorexia nervosa
hx of seizures
heavy alcohol use

Clomipramine is a TCA that _________ the seizure threshold

lowers

What can cause phalangeal hypoplasia in 2.5kg baby girl whose mom has a past medical history of epilepsy

Phenytoin

Short limbs and phalangeal hypoplasia

teratogenicity caused by phenytoin

Higher doses of ______ are recommended for women taking anti-epileptics such as phenytoin or valprate

folate

These medication are excreted unchanged in urine

Desvenlafaxine
Milnacipram

Venlafaxine and desvenlafaxine are ______% protein-bound

27%

Which potential neonatal risk is associated with continuing antidepressants during pregnancy

discontinuation syndrome (seen in higher rates with paroxetine)

Only medication approved by FDA for the treatment of bulimia nervosa

Fluoxetine

Infantile anorexia interferes with which important developmental event

somatopsychological differentiation

When an infant’s feeding is directed by his or her emotional needs instead of physiological sensation of hunger and satiety, he or she fails to develop

somatopsychological differentiation

the ability to differentiate sensations in the body (results in infant’s emotional needs (affection, dependency, anger, and frustration) rather than hunger needs dictating behavior

somatopsychological differentiation

Posttraumatic feeding disorder (PTFD) usually has a ____ onset after________

sudden
choking or insertion of a gastric tube

Which of the following is a behavioral therapy technique for anorexic or bulimic patients who binge and purge?

Have a supervisor watch them for 2-3 hours after every meal because they rarely feel conf

Studies suggest that family therapy is most helpful in which population of anorexia nervosa patients

younger patients with shorter illness

Anorexic pt asks for treatment for her severe constipation. What should be administered

docusate

An NP is interviewing a client who is incapacitated by a fear of public speaking who must give presentations as part of her job. Which non-pharmacological treatment is most likely to be effective to resolve this problem?

Behavioral therapy (Systemic desensitization)

systemic desensitization therapy is a a type of behavioral therapy is effective for eliminating _____?

phobias
OCD
anxiety disorder

Systematic desensitization therapy consists of

relaxation training
hierarchical construction of situations in order of increasing anxiety
desensitization of the stimulus

Brief psychotherapy that focuses on resolving interpersonal problems

interpersonal psychotherapy

Interpersonal psychotherapy takes ______ to complete

12-16 weeks

Psychotherapy used for people struggling with longstanding difficulties in the way they think and feel about themselves, their environment, and other people

psychoanalysis

The principles that govern behavioral therapy are based on early studies of _______ by _______ and _________ by ________

operant conditioning
Skinner

classical conditioning
Pavlov

In patients with eating disorders, which characteristics do not require hospitalization

nausea/vomiting

Main indications for hospitalization in pt with eating disorder

severe starvation and weight loss
hypotension
hypothermia
electrolyte imbalance
suicidal ideation/psychosis
failed outpatient tx

Median age of onset for generalized anxiety disorder

early adulthood (25-35)

What is the typical age of onset for panic disorder?

Early adulthood

What treatment can cause cardiac malformations in the developing fetus if used in a pregnant patient

Lithium

Standing in line at a store would likely provoke anxiety in patients with

agoraphobia

Which medical condition is associated with anorexia nervosa

Lanugo
bradycardia
pancytopenia
osteopenia
metabolic encephalopathy
arrhythmias
elevated LFTs
elevated BUN
decreased T3/T4
parotid gland enlargement
seizures
peripheral neuropathy

What is the most common personality disorder in bulimia nervosa patients

borderline

Children with PTSD have nightmares involving______ rather than recurring dreams of traumatic events

monsters
rescues
threats

The DSM-5 diagnosis of anorexia nervosa no longer requires

irregular menses in postmenarcheal females
(unspecified eating disorder replaces NOS)

Fluoxetine has a half life of approximately

48 hours

Venlafaxine has a half life of

5 hours (parent drug)
11 hours (active metabolite)

Citalopram has a half life of approximately

24-48 hours

Sertraline has a half life of approximately

26 hours

Of the following: amitriptyline, duloxetine, mirtazapine, and sertraline, which has the fewest side effects

Mirtazapine

The risk of weight gain with mirtazapine may be diminished at doses

>30mg/day

The noradrenergic effects of mirtazapine tends to be

dose dependent
increases significance at >15mg/day

noradrenergic

activated by, or involving norepinephrine in the transmission of nerve impulses

Mirtazapine blocks serotonin 2A, 2C, and 3 receptors, so the side effects of _______ are avoided

anxiety
nausea/Gi distress
sexual dysfunction

In Mirtazapine, blocking serotonin 2A and H1 antihistamine receptors accounts for the side effects of

sedation
weight gain

Mirtazapine appears to have no clinically significant effect on

seizure threshold
cardiovascular syndrome

A dose dependent relationship between Miratazpine and _____ has been demonstrated

increased appetite and weight

Unlike DSM4, in DSM 5, which of the following statements is true regarding separation anxiety disorder?

It is diagnosed with onset at any age.

Separation anxiety disorder last for _____ in children or _______ in adults

4 weeks
6 months

Among the general population, which of the following disorders is most commonly comorbid in anorexia nervosa

Major depressive disorder (56%)

25% of anorexic pts have these disorders

alcohol use disorder
specific phobias
social anxiety disorders

Up to 35% of anorexia patients have

obsessive compulsive disorder

Double the dose of lamotrigine when taking the following medications concomitantly

carbamazepine
phenytoin
phenobarbital
primidone
rifampin

The lifetime prevalence of social phobia is _____ and slightly more common in______

13%
women

What ethnic group is at the highest risk of having social phobia?

Native American

People who have social phobia are less likely to live in ______ setting

an urban

Onset of social phobia is usually in

adolescence
early adulthood

FDA approved medication for social phobia

paroxetine
(beta-blockers public speaking)

The 3 CBT techniques used for social phobia treatment are

exposure
cognitive restructuring
social skills training

Subtypes of social phobia

Natural environment (heights, lightning, bodies of water)

Blood injection injury (fears of blood medical procedures, needles etc)

Situational (fears of situations such as being in an elevator tight space, driving or airplanes)

What social phobia subtype is thought to run in families

Blood-injection injury
associated with strong vasovagal response

the feeding disorder termed “state regulation” is defined by which of the following descriptions

infant cannot feed adequately due to increased irritability/crying or excessive sleepiness

Potassium at this level is grounds for involuntary admission for eating disorder in adults

<3.0

HR ____ is ground for involuntary admission for eating disorder in adults

<40

Glucose _______ is grounds for involuntary admission for eating disorder in adults

<60

BP ______ is grounds for involuntary admission for eating disorder in adults

<90/60

For children and adolescent, involuntary admission for eating disorder include

HR near 40bpm
orthostatic (>20bpm increase HR or >10-20mmHg BP drop)
BP<80/50mmHg
hypokalemia
hypophosphatemia
hypomagnesemia

Temp _____ is grounds for admission for eating disorder in adults

<97 degrees

How long must symptoms be present for a patient to be diagnosed with a specific phobia

6 months

Neuropsychological tests evaluate

cognitive factors

Clinical psychological tests evaluate

emotional factors

Pts with ____ are known to have impaired neuropsychiatric function

schizophrenia
Panic Disorder
OCD
PTSD

FDA approved for tx of social anxiety disorder

Paroxetine
Sertaline
Beta blockers
long-acting form of venlafaxine

People with social phobia often have comorbid

GAD
PTSD
Separation anxiety disorder
MDD
bipolar
Alcohol dependence

Pt 78% of her ideal body weight for height. Amenorrheic for several months. Admits to using laxatives and diuretics. What metabolic abnormality would you expect to see?

Hypokalemic alkalosis

Diarrhea normally produces what metabolic abnormality

hyperchloremic metabolic acidosis from bicarbonate losses

Diarrhea associated with chronic laxative abuse is risk in

hypokalemia and saline (chloride)-responsive metabolic alkalosis (increased renal-bicarbonate reabsorption)

Excessive anxiety and worry must occur for at least this period of time to make a GAD diagnosis?

6 months

The lifetime prevalence rate of GAD is

5%

The duration of disturbance of PTSD must take place for at least in order to make a diagnosis

1 month

The lifetime projected risk of PTSD is _____ with symptoms varying over time

8.7%

A history of _____________ in first degree relatives is related to increased vulnerability for developing PTSD

depression

In order to be diagnosed with panic disorder, the change in behavior resulting from attack must last for at least

1 month

The prevalence of panic disorder is

2.7%

First degree relatives of patients with panic disorder are up to _____x more likely to develop panic disorder

8

in vivo exposure is also sometimes called

flooding

Anorexia nervosa is most common in which of the following socioeconomic classes?

No 1 association

Anorexia nervosa is more common in young women in these professions

modeling
ballet

What is the 12-month prevalence of panic disorder among adults and adolescents in the US?

2-3%

Benzodiazepines are more effective than SSRIs in treating _________ of anxiety

physical symptoms

What is the most common category of mental illness in the United States

Anxiety disorder

Function MRI measures which of the following

Changes of blood flow in brain

TSH is low in

hyperthyroidism

Hyperthyroidism may manifest like ______ before classic sx of Graves’ disease appear

generalized anxiety
or
panic

In patients presenting with dementia, what is true about brain tumors?

paraneoplastic limbic encephalitis should be suspected

Early sx of paraneoplastic limbic encephalitis are

confusion
agitation
memory loss progressing to dementia

Tumors tend to be ______ symptomatic than ischemic strokes affecting comparable brain volume

less

Brain tumors in adults are associated with psychiatric symptoms in ________ of patients

50%

What is the superior level of evidence in regards to therapeutic clinical research?

RCT, especially meta-analysis of multiple RCT’s

Imaging of Cushing’s disease requires an ____ because it is commonly caused by _____

MRI
pituitary adenoma

Cushing’s syndrome is often preceded by

psychiatric symptoms:
-irritability
-crying
-pronounced suicidality
-rarely psychosis

When Cushing disease pts require high doses of steroids, it may trigger

mania, requiring mood-stabilizing medication as prophylaxis

FDA approval for Marinol (dronabinol)

Anorexia in AIDS
anti-nausea/antiemetic for chemo

Once rabies reaches the CNS, a pt will experience

physical agitation
episodic confusion
psychosis
combativeness

In rabies, the virus travels

inwards from the peripheral nerves to infect the CNS
Nonneuronal tissues can be affective

Paralytic rabies occurs in _____ of pts exposed to rabies

20%

Vocal cord paralysis often occurs in

rabies

Hypothyroidism carries a greater risk of developing

carpal tunnel syndrome

Depressive symptoms persist in hypothyroid pts after therapy in _____% of pts

10

A manifestation of late state hypothyroidism

myxedema madness

myexedema madness

hallucinations and paranoia
late state of hypothyroidism

physical findings in hypothyroidism

thin dry hair
dry skin
constipation
carpal tunnel syndrome
lateral eyebrow loss
hearing loss

When studying pt’s favorite color and presence of bipolar disorder, which test will analyze the data most effectively?

Chi-square test

Used to compare samples with non-measurable nominal/categorical variables

chi-square test

used to compare the means of 2 independent and different sample populations (ie height of men in Texas and NY)

T-test

It requires a large sample or known population variance

z-test

Study follows a group of similar individuals to determine risk of developing disease

Cohort Study

Analyzes a known outcome by looking back in time to assess exposure

Case-control

Analyzes a population (or representative subset) at a single point in time (a cross sectional) to assess disease incidence and prevalence

Cross sectional study

Experimental/clinical trial that studies the effect of an intervention by comparing the treatment to a placebo in 2 equivalent groups separated by randomization

RCT

In patient with cerebral ataxia, Parkinsonism, bone pain, mental slowing, attention problems, hypercalcemia and elevated parathyroid hormone, what is true

Correcting calcium level will prevent delirium

Stones (kidney stones), groans (bone pain), and psychiatric overtones is mnemonic for

hypercalcemia

As many as _____ % of patient with hyperparathyroidism and elevated calcium have no recognizable symptoms

50%

Which ethic group tends to have the highest preference for life-sustaining tx in terminal illness?

African Americans

What ethnic group has the lowest rate of preference for life-sustaining treatment in terminal illness

European Americans

The most common value in a data set is called the

mode

An estimate _____% of pts with chronic alcohol use disorder relapse within 3 months after treatment completion

50

Which of the following medications is not recommended for relapse prevention in detoxified patients with chronic alcohol use disorder

Chlordiazepoxide

Anti-craving drugs used to prevent alcohol relapse

Topiramate
Acamprosate (also works in acute post-withdrawal period)

For an idealized population graphed into a perfect bell shaped curve, 1 standard deviation of the mean would include what percentage of people?

68%

For an idealized population graphed into a perfect bell shaped curve, 2 standard deviations of the mean would include what percentage of people?

95%

For an idealized population graphed into a perfect bell shaped curve, 3 standard deviations of the mean would include what percentage of people?

99.7%

Tumors of the upper brainstem can cause

akinetic mutism

Early signs of encephalitic paraneoplastic syndromes are

confusion
agitation
memory loss progressing to dementia

Leading cause of paraneoplastic syndromes

small cell carcinoma of the lung

Cancer that is frequently preceded by depression

Pancreatic

New onset of symptoms and _____ increase the likelihood that psych sx may be due to HIV related cause

CD4 count<600 (generally seen only in advanced disease stages)

What is true about acute intermittent porphyria (AIP)

phenothiazines are safe to treat psychosis

In acute intermittent porphyria (AIP) what medications should be avoided

phenytoin
barbiturates
valproic acid

The onset of acute intermittent porphyria (AIP) is between

20-50 years old

Classic triad of acute intermittent porphyria are

1. episodic acute colicky abdominal pain
2. motor polyneuropathy
3. psychosis

What medications can promote porphyrin synthesis?

meprobamate (anxiety med/tranquilizer)
sulfonamide antibiotics
ergot derivatives
many anti seizure meds
HIV drugs
hormonal contraceptive
anesthetics

analysis of variance (ANOVA) is used to

compare the means of multiple groups to determine if all are equal

A paired t-test is used to

compare the means of 2 related sample populations (ie disease metric in same population before and after treatment, twin study)

With partial seizures _______ have normal EEG

40%

Partial seizures often derive from

deep limbic brain structures (commonly temporal lobe)

Partial seizures are responsible for _______ of the nonconvulsive seizures and are under-diagnosed

60%

Over ________ epileptic patients experience depression with higher rates in patients with

50%
complex partial seizures

Used to compare the means of 2 independent sample populataions

Student’s t-test

lesions with a predilection for the temporal and inferomedial frontal lobs are specific for encephalitis produced by

herpes simplex virus

Sensitivity of 95% implies that

5 percent of the tests are false negatives

Specificity of 95% means that

5 percent of the tests are false positives

Depression in adolescents is likely to present with ________ rather than mood change

irritability

To study cigarette smoking as a risk factor for the development of schizophrenia, a study is planned to survey 100 pts with schizophrenia and 100 age-matched controls with regards to their smoking hx. Which type of study is this?

Case-control study

Pt with chronic alcohol use disorder presents with ataxia, confusion, memory loss & seizures. Pt was given a glucose bolus. After a few hours he develops diplopia, horizontal nystagmus, lateral rectus palsy, diminished deep tendon reflexes and a positive Babinski’s sign on the left side. What caused these symptoms?

Wernicke’s encephalopathy

What is often seen on MRI in Wernicke’s encephalopathy

bilateral hyper intensity of mammilary bodies

Wernicke’s encephalopathy progresses when

glucose is given without thiamine

Which of the following descriptions correctly identifies clonidine’s role in withdrawal from opioids?

Binds alpha 2 autoreceptors, decreases locus coeruleus cell firing, and reduces noradrenergic release

Clonidine does not reduce the _______ involved in opioid withdrawal

cravings
lethargy
insomnia
restlessness
muscle ache (pain medication may be needed during taper)

During opioid withdrawal, the dose range is

0.6-2.0mg /day (not FDA approved)

Breaking large group into smaller groups, with each more homogenous by some measure

Cluster analysis

Statistical technique that seeks to explain the variance of multiple variables by utilizing a smaller number of unobserved factors

Factor analysis

Process of reducing the number of variables studied to focus on a smaller number

Dimension

Used to determine independent variables (X) that appear to affect the quantity of interest (Y)

Discriminant analysis

For psychiatric patients who are >50 years of age, any change in mental status, mood, personality or behavior warrants an

MRI examination

Weight loss with no other explanation could indicate

malignant disease

Acute onset of behavioral and personality changes in a pt with no hx of psychiatric disorders could indicate

frontal lobe syndrome caused by a brain tumors that could go undiagnosed with a CT. Needs a MRI to dx.

What percentage of opioid-addicted patients relapse 12 months after undergoing successful detoxification?

>80% even for the most motivated patients

occurrence of confusion, altered LOC and coma as a result of liver failure

Hepatic encephalopathy

Focal neurological signs are less common in cirrhotic ______

men

Episodic disorientation, rage or inappropriate behavior in chronic alcoholics may precede

progressive decline in cognition, memory and speech

Classic findings of liver failure include

ascites
jaundice

Earliest signs of liver failure

mild intellectual deficits

Term for alcoholics used in DSMIV that were eliminated in DSM-5

Alcohol Dependence

A psychiatrist shows pt 3 1-dollar bills and hides the bills while she watches. Pt is unable to locate the bills after several minutes. A problem in what area best explains her deficits?

Recall

Memory of riding a bike or cognitive skills like reading

Procedural

Memory of autobiographic events that can be consciously assessed and explicitly stated

Episodic

Chronic alcoholic presents to ED with hypothermia, tachycardia, ataxic gait with foot weakness and foot drop. What is the most likely diagnosis?

Wernicke’s encephalopahy

Treatment for Wernicke’s Encephalopathy

Thiamine (vitamin B1)

Chronic alcoholic presents with amnesia, confabulation and apathy

Korsakoff’s syndrome

Combining 3 variables into single variable to describe a term that encompasses the 3 variables that might not be able to measured directly but can be inferred by the the affect of the 3 variables.

Factor analysis

Measure of the relationship between the change of 2 variables

covariance

Pt attempted suicide by sitting in running car parked in closed garage. He’s unresponsive, where might an MRI demonstrate lesions?

Diffusion-weighted of the globbus pallidus and deep cerebral white matter

Signs of carbon monoxide poisoning

polycythemia
nausea
dizziness/seizures
headaches
confusion/memory loss
impaired consciousness/coma
cherry-red skin discoloration
cyanosis
dyspnea
CP

5/50 pts with BPD attempt suicide 5/500 of control group attempts suicide. What calculation determines the suicide risk attributable to BPD?

(5/50)/(5/500)

5/50 pts with BPD attempt suicide 5/500 of control group attempts suicide. What calculation determines the relative risk attributable to BPD?

(5/50)/(5/500)

Samples of subjects who represent the larger population

may allow estimations of error in resulting statistics

are often more accurate than those of a population

can be studied more quickly at a lower cost

Scoring someone on performance slightly higher that it would normally be score because the researcher knows the participant is not performing at his best (based on participant’s past performance) is called

Halo effect

When a participant changes his or her normal patterns of behavior when he or she is a member of a study or is being watched or observed

Hawthorne effect

Causes hospital cases and control cases to be systematically different because the combination of exposure to risk and occurrence of disease increases the likelihood of being admitted to the hospital

Berksonian Bias

Peak timing after last drink for the occurrence of delirium tremens is

2-3 days

Vital signs on pt developing delirium tremens usually are

HTN
tachycardia
febrile

Pt presents with palpitations, lid lag, weight loss, thyroid bruit, and thyroid mass/enlargement

thyrotoxicosis

Pt with abnormal temperature, pulse, respiratory rate, or blood pressure and diaphoresis most likely has

autonomic abnormality

repetitive non-goal directed motor activity, not inherently abnormal but repeated frequently

stereotypy

Pt with visual hallucinations has the following lab work. Pancytopenia with megaloblastic changes: MCV 118, Pat 96,000, WBC 2800. Folate WNL. What will help with the hallucinations

Vitamin B12 shot

phenomenon in meta-analyses in which 2 studies with similar correlations can be combined to exhibit the opposite trend

Simpson’s paradox

observed when individual response most strongly to a seemingly dangerous experience when faced with it for the first time.

Novelty effect

phenomenon that people tend to perform to the expectations placed on them

Pygmalion effect

analyze a population at a single point in time to assess disease incidence and prevalence

cross-sectional study

________ is suggestive of late Alzheimers disease

Cerebellar atrophy

adult pt with myoclonic jerks, rapid progress towards dementia, and akinetic mutism with specific EEG findings

Creutzfelt-Jakob disease (CJD)

Occurs more often in children after previous measles (rubeola) infection

Subacute sclerosing panencephalitis (SSPE)

14-3-3 protein in CSF may be the most discriminating evidence for

Creutzfeldt-Jakob disease (CJD)

Creutzfeldt-Jakob disease (CJD) affects patients aged

50-70

32 year old F presents with fatigue, sleep disturbance, HA, blurred vision, and depressed mood for 2 months. Her hx notable for anemia requiring multiple transfusion. She has butterfly shaped rash around eyes and on cheeks. Also c/o intermittently aching joints and diarrhea. Which statement about disease is true?

Disease symptoms can be mistaken for somatization

2 year old F presents with fatigue, sleep disturbance, HA, blurred vision, and depressed mood for 2 months. Her hx notable for anemia requiring multiple transfusion. She has butterfly shaped rash around eyes and on cheeks. Also c/o intermittently aching joints and diarrhea. Clinical presentations of this patient suggests

lupus erythrematosus

Age of onset of symptoms of lupus is usually from ______

20-50

When is the typical onset of panic disorder in terms of the first panic attack?

First panic attack often occurs “out of the blue” while doing everyday things

longitudinal study in which subjects receive different treatments over the course of the study, typically so that all subjects receive the same exposures

crossover study

What are the most common causes of intellectual disability?

Fragile X
Downs syndrome
Fetal alcohol syndrome

What percentage of adolescents with MDD will develop Bipolar I

10-15%

What is the most common inherited form of intellectual disability in males

Fragile X

It is associated with FMRI gene on the X chromosome

Fragile X

Most common acquired form of intellectual disability

Fetal alcohol syndrome

Prevalence of intellectual disability in the community is between

1-3%

Most common genetic form of intellectual disability

Down syndrome (trisomy 21)

Mixed episodes of bipolar 1 are more common in

adolescents
women

Concerta is _____% IR and ____% ER

22
78

Metadate CD is _____% IR and ____% ER

70
30

Ritaline LA is _____% IR and _____% ER

60
40

Prevalence of alcohol abuse and dependence in bipolar patients

46% (compared to 13% in general population)

Individuals with OCD and intellectual disability have

increase anxiety with response-blocking intervention

Who will have increased anxiety with response-blocking intervention

pts with OCD and intellectual disability
pts with stereotypy

In patients with intellectual disability, if there is an acute change from baseline behavior, what needs to be considered

trauma
posttraumatic stress
(population is particularly vulnerable to exploitation by others)

To be dx with autism spectrum disorder, the child must have disturbances in which of the following domains

social relatedness
communication
restricted interests and activities

What period dramatically increases the risk of recurrence in female pts with bipolar disorder

postpartum

The DSM-5 now states that symptoms must be present before age of ______ in ADHD

12

Ritalin LA is ______% IR and _____% ER

60
40

In DSM-5 absence of ____ is captured in a specified with body dysmorphic disorder

insight

Clonidine is particularly helpful in pts with these comorbid disorders

ADHD
tic disorder
conduct disorder or oppositional defiant disorder

At low doses clonidine is associated with

the stimulation of inhibitory presynaptic auto receptors in the CNS

_________ is helpful for ADHD pts with ADHD-associated sleep disturbances and may be helpful for reducing anxiety and hyper vigilance in traumatized children

Clonidine

Pts with _____ and _____ may need their lithium dose held to avoid toxic levels

diarrhea
vomiting

Which medication has demonstrated efficacy in the management of chronic anxiety in patients with severe lung disease and may assist with improvement in respiratory status (improves exercise tolerance)

Buspirone

Have been reported to result in interstitial lung disease

Venlafaxine SNRI
also SSRIs

This type of antidepressant is not recommended in pts with difficulty clearing secretions (anticholinergic effects)

TCA

Modafinil is FDA approved for

narcolepsy
shift work disorder
excessive tiredness from obstructive sleep apnea

Diminished and confabulatory self reporting is characteristics of patients with

intellectual disability

The Star D study found that ______% of pts taking citalopram had decreased libido

54

The Star D study found that _____% of males taking citalopram had erectile dysfunction

37

The Star D study found that ___%_ of pts taking citalopram had difficulty achieving orgasm

36

Separation anxiety is a normal developmental process commonly seen at ________ of age

10-18 mo

Stranger anxiety is typically seen at around ______ of age

8 months

______ nerve carries motor fibers to most of the extra ocular muscles of the eye

oculomotor (CN III)

elevates upper eyelid

levator palpebrae superioris (CN III)

elevates eyeball

superior rectus (CN III)

moves eyeball medially (adduction)

medial rectus (CN III)

moves eyeball downward

inferior rectus (CN III)

moves eyeball superiorly and laterally

inferior oblique (CN III)

moves eyeball inferiorly

superior oblique (CN IV)

eyeball adduction

lateral rectus (CN VI)

carries visual fibers only

Optic Nerve (CN II)

innervates superior oblique muscles which moves eyeball downwards and slightly laterally

Troclear nerve (CN IV)

innervates the lateral rectus, moving the eyeball to the side

Abducens nerve (CN VI)

Escape-avoidance is an example of a ________ behavior type

functional

Four functions that maintain behaviors

Escape/Avoidance
Attention Seeking
Materials
Sensory Stimulation

Individual behaves in order to get preferred item or participate in an enjoyable activity

Materials

Individual behaves to get focused attention from parents, teachers, siblings, peers or other people that are around them

Attention seeking

Individual behaves in a specific way because it feels good to them

Sensory stimulation

Functional behaviors are generally not driven by_____
and are not always employed with _________

underlying psychiatric conditions

direct conscious awareness

Patients have short stature with hypogonadism, hyperplasia, and obesity

Prader Willi syndrome

The genetic abnormality in 70% of Prader Willi syndrome patients is a ________________

deletion at chromosome 1

Most associated psychiatric disorder associated with Prader-Willi syndrome

OCD

Thoughts, impulses, and behaviors that are felt to be repugnant, distressing, unacceptable or inconsistent with one’s self-concept.

Ego-dystonic

Inadequacies are only recognized in other and the external environment and pts do not harbor ego dystonia or question themselves

OCPD

Thoughts are intrusive, ego-dystonic and distressing to individuals, drastically interfering with everyday life

OCD

What is the main value of selegiline patch?

Bypasses gastic tract, obviating the need for dietary restriction at lowest dose

Prevalence of autism globally

1 in 100

Seeking out experiences that are consciously or unconsciously fearful. Ie experimenting with drugs, attraction to horror movies

Counter phobia

Expression of unconscious conflicts in a setting other than the one in which they initially arose

Acting out

In the criteria for premenstrual dysmorphic disorder, at least _____ out of ______ specific symptoms must be present during _______

5
11
final week before onset of menses

In Downs syndrome patients ________ may mimic psychiatric disorders

Thyroid problems (hypo)

In Down syndrome patients, dementia symptoms often begin in patients in their _________

40s and 50s

Self-injury is characteric of patients with _____________ syndrome

Lesch-Nyhan

In Fetal alcohol syndrome pts have anomalies in the _______

premaxillary zone

Flat upper lip
Flattened philitrum
Flat midface

Fetal alcohol syndrome

Upslanting palpebral fissures
Flat nose bridge
Suggestion of epicentral folds

Down syndrome

Broad forehead
Short nose with broad tip
Full cheeks
Wide mouth with full lips

Willis syndrome

Narrow forehead
Almond-shaped eyes
Triangular mouth

Prader-Willi

What is the most common comorbid psychiatric disease in epilepsy?

Depression (7.5% in intractable pts as high as 50%)

What percentage of bereaved people meet criteria for MDD?

20%

Acute grieving period should last no more than

6 months

The reduction in tics and ADHD is associated with guanfacine at what doses for school age children?

0.5mg bid to 1mg TID

Guanfacine is associated with less _______ and more __________ than clonidine

sedation
duration of action

Guanfacine is associated with minor, clinically insignificant

bradycardia and hypotension

Guanfacine is a more __________ compound than clonidine

selective alpha-adrenergic

The first episode in both women and men with bipolar I is usually

major depressive

The number of manic episodes usually ___________ compared to depressive episodes in men

equals or exceeds

The number of manic episodes usually ________ compared to depressive episodes in women

equals or is less

What percentage of individuals who are diagnosed with specific learning disorder with impairment in reading are male?

60-80%

Older individuals who are depressed constitute a disproportionate number of

suicides (13% of poplulation, but completed rate is 20%)

Depressed older individuals display more

neurovegetative signs
cognitive disturbances

About _______% of people with CVD develop depression

>=25

Coprolalia

involuntary uttering of obscenities

Coprolalia is present in what percentage of patients with Tourette’s disorder?

10

How long must symptoms be present in adults to be diagnosed with cyclothymic disorder

2 years

When treated with antidepressants, depressive symptoms usually resolve in an expected pattern. What is the predictable progression of symptom resolution

Side effects occur
Neurovegetative symptoms resolve
Mood improves

After the first psychotic _____% of schizophrenic patients will experience a benign course, in contrast to the rest who fail to recover or are hospitalized in the first 2 years after a first hospitalization for psychosis

33

When compared to children of non-depressed mothers, infants and children of depressed mothers display which of the following?

Increased fussiness (more easily upset)
less facial expression
less crying
less head orientation
ineffective emotional regulation as they age
slowed motor development by 6 months
poorly integrated reactions
lower self-esteem beginning in preschool

Which neurotransmitter has the most evidence for its association with OCD

Serotonin

Pts with OCD have _______ hyper function in the ___________ and _______hypofunction in the ____________

dopaminergic
PFC (mesocortical dopamine pathway)
serotonergic
basal ganglia

FDA-indicated (age 6-17) maximum dose of dexmethylphenidate (Focalin)

20mg

FDA-indicated (age 6-17) maximum dose of Metadate (ER/CD)

60mg

FDA-indicated (age 6-17) maximum dose of Methylphenidate-OROS (Concerta)

72mg

FDA-indicated (age 6-17) maximum dose of Methylphenidate patch (Daytrana)

30mg

FDA-indicated (age 6-17) maximum dose of Methylphenidate LA

60mg

FDA-indicated (age 6-17) maximum dose of Methylphenidate SR

60mg

Collaborative care occurs in

primary care settings (primary care and mental health professionals working closely to deliver effective tx for depression and other common psychiatric disorders)

What dementia has the lowest incidence of depressive sx

FTD
Pick’s disease

_________ is commonly seen in FTD and often mistaken for depression

apathy

Early Sx of FTD

Disinhibition
Hyperorality
Compulsive Behaviors

Mood disorders common in pts with Parkinson’s disease

depression
anxiety
apathy/abulia

_______ offered an early neuroprotective effect in Parkinson’s disease patients, but lacks long term benefit

Selegiline

Temporary decreases amount of levodopa and carbidopa needed to control Parkinson’s symptoms

Selegiline

Antidepressants as a class can worsen symptoms of

Parkinson’s disease

Long-term _______ use may have a neuroprotective effect in individuals with Parkinson’s disease

Lithium

Child has reduced or absent reticence in approaching and interacting with unfamiliar adults, overly familiar verbal or physical behavior not consistent with culturally sanctioned/age-appropriate social boundaries, diminished or absent checking with an adult caregiver after venturing away, and willingness to go off with an unfamiliar adult with minimal or no hesitation. Child has experience pattern of extremely insufficient care (especially age 0-3years)

disinhibited social engagement disorder

What part of the brain controls the respiratory functions of the body

Medulla

The medulla is located in the

brainstem

The part of the brain contains sensory and motor functions

cerebral cortex

includes the function of connecting the upper and lower parts of the brain and messaging between those parts

Pons

coordinates body movements and balance and coordination

Cerebellem

Which of the following modalities should be avoided when treating psychiatric disorders in pts with intellectual disability (formerly mental retardation)

Potent anticholinergic medications

generally happy, active children from birth and adjust easily to new situations and environments

Easy children

generally mellow, less active babies from birth, and have some difficulty adjusting to new situations

Slow-to warm children

Have irregular habits and biological routines (ie eating, sleeping), have difficulty adjusting to new situations and often express moods very intensely

Difficult children

3 general types of temperaments according to Thomas and Chess

easy
slow-to-warm
difficult

The 9 dimensions of temperament as described by Stella Chess and Alexander Thomas

Attention span and persistence
Adaptability
Approach or withdrawal
Activity level
Rhythmicity
Distractibility
Intensity of reaction
Threshold of responsiveness
Quality of mood

The length of time the child pursues a particular activity without interruption and the child’s persistence in continuing activity despite obstacles

Attention span and persistence

Child’s long term (as opposed to initial) response to new situations

Adaptability

Nature of child’s initial response to a new situation or stimulus

Approach or withdrawal

Oedipal complex describe feelings that start during the ________ phase

phallic (Freud)

boy’s unconscious desire to possess his mother sexually and to kill his father

Oedipal complex (Freud)

Penis envy occurs during the ________ phase

phallic (age 3-5, Freud)

_______refers to boys aged 3-5 who have feelings of guilt regarding the oedipal complex

castration anxiety

Freudian phases that occurs from birth to 18 months

Oral phase

Freudian phase that occurs from 18 months to 3 years

Anal phase

Freudian phase that occurs between 3-6 years

Phallic phase

Freudian phase that occurs between 6 years to puberty

Latency Phase

During this Freudian phase, much of the child’s energy is channeled into developing new skills and acquiring new knowledge, and play becomes largely confined to other children of the same gender

Latency Phase

Freudian phase between puberty and adulthood

Genital stage

In Freud’s _____ phase, sexual instinct is directed to heterosexual pleasure, rather than self-pleasure like during the ______ phase

genital
phallic

Symptoms of sexual abuse include

imitate sexual acts on others
somatoform symptoms
bedwetting
eating disorders
talk excessively about sexual acts

This part of the brain plays a key role in long-term memory, hearing, and interpretation of visual stimuli

Temporal lobes

What is the primary neurotransmitter associated with anxiety and panic attacks

GABA

Primary inhibitory neurotransmitter in the nervous sytem

GABA

Neurotransmitter implicated in Alzheimer’s dementia and is widespread throughout the CNS, modulating musculoskeletal response

Acetylcholine

Endogenous opioids that modulate pain

Beta endorphins

Excitatory neurotransmitter in the CNS

Glutamate

Twin studies have estimated the heritability of reporting any suicidal ideation, suicidal plans or attempts to be about ______ and that of reporting suicide attempts to be _______

44%
55%

Eating disorder that involves eating items that are not typically thought of as food and that do not contain significant nutritional value, such as hair, dirt, and paint chips. (not culturally supported or socially normative practice)

Pica

1st line treatment for pica

testing for mineral or nutrient deficiencies, and correcting those

2 of the most common causes of pica

anemia
malnutrition

Ingesting kaolin (clay) is a culturally sanctioned practice in certain _______ cultures

African

Children presenting with sx of OCD also show substantial comorbidity in family studies for (shared genetic diathesis):

Eye blinking and throat clearing
initial motor and vocal tics

Erikson’s Theory stages of development in correct sequence

Trust vs mistrust (birth -18mo)
Autonomy vs shame and doubt (18 mo-3 years)
Initiative vs guilt (3-5 years)
Industry vs inferiority (6-12 years)
Identity vs role confusion (12-18 years)
Intimacy and solidarity vs isolation (18-35 years)
Generatively vs self-absorption & stagnation (35-55 or 65)
Integrity vs despair (55 or 65 – death)

Discrete trial training (DTT) is used with applied behavioral analysis (ABA) in

school programs for children with pervasive developmental disorder

Systemic approach to develop a program for each child by evaluating a child’s strength and weaknesses, identifying educational and treatment goals, and applying techniques for skill development

Applied Behavioral Analysis (ABA)

_________ reinforcement is used in Discrete trial training (DTT) and used to teach ________

Positive
new skills

Since children have a higher metabolic capacity, drugs must be given at

shorter intervals (ie neuroleptics, tricyclics, and methylphenidate)

Children excrete drugs like lithium faster than adults because they have

more efficient renal elimination

Since children have lower adipose tissue, how does this affect dose

results in a larger concentration of lipophilic drugs when given the same weight-adjusted dose as adults

How does children’s greater volume of extracellular water affect drugs like lithium

has lower plasma concentration compared to adults.

Critics view Lawrence Kohnberg’s stage theory of development as

Western
predominantly male
hierarchical

Kohlberg’s theories are an expansion of:

Piaget’s theories on cognitive and moral development

Kohlberg describes ______ stages of moral reasoning determined by __________ rather than ____________

6
a person’s thought process
moral decision reached

Behaviorally inhibited children are more likely to experience

anxiety, depression, and phobias

Children who are_______________ tend to have poor peer relations and get in trouble with the law. They are difficult partners and roommates.

negative, impulsive, and unregulated

auditory processing difficulties may be aggravated by

low sensitivity

memory problems may be intensified by

high distractibility

The single most important risk factor for preschool mental health problems is

difficult temperament

A 30 year old male often has difficulty expressing himself due to frequent incidences of getting “stuck” on a syllable while trying to speak. The patient often feels anxious when required to give a presentation at work. This problem became apparent when he was in the 2nd grade. What is the most appropriate DSM-5 diagnosis?

Childhood-onset fluency disorder

DSM-5 dx for stuttering

Child-onset fluency disorder

The term “mental retardation” used in DSM-IV has been replaced by this in DSM-5

Intellectual disability

IQ score of pts dx with intellectual disability

2 standard deviations below population or 70 or bellow

Percentage of population with severe intellectual disability

3-4%

In children with schizophrenia, akathisia may be confused with

ADHD
agitation

Clozapine is typically used after at least _______ adequate trials of other antipsychotics fail to resolve psychotic symptoms

2

Extrapyramidal signs occur in up to _______ of pediatric patients on first generation antipsychotics

75%

Acute episodes of EPS in pediatric patients can be treated with

anticholinergic agens
antihistamines
amantadine

The most frequently studied genes implicated in the development of substance use disorders encode which substance

Aldehyde dehydrogenase

Gain-of-function polymorphisms of the SLC6A4 promoter have been linked to

OCD

Converts L-DOPA to dopamine

DOPA decarboxylase

Rate-limiting step in serotonin synthesis

tryptophan hydroxylase

Ratio male:female ADHD diagnosis in children

2:1

Ration male:female ADHD diagnosis in adults

1.6:1

Prevalence of ADHD in children

9-12%

_______ takes place when a reinforcement or punishment is used to modulate the frequency of a particular behavior

Operant conditioning

What is the prevalence of child-onset fluency disorder in the population of children and adolescents?

5%

What is the sequence of infant development described by Margaret Mahler

Autistic phase
Symbiotic phase
Separation-individuation

Autistic phase occurs during the

first few weeks of life

Phase characterized by total detachment and self-absorption in which the infant spends most of his or her time sleeping

Autistic

The Symbiotic phase occurs between

a few weeks of life and 5 months

During this phase, the infant recognizes his or her mother but lacks a sense of individuality

Symbiotic

Separation-individuation, according to Margaret Mahler, has these 4 subphases

Hatching (5-9 mo)
Practicing (9-16 mo)
Rapprochement (15-24 mo)
Object constancy (24-36 months)

Infant shows increased interest in the outside world

hatching (1st phase of separation-individuation)

infant develops the physical ability to separate from mother

practicing (2nd phase of separation-individuation)

infant explores the outside world but requires the mother to be present and to be able to support the child emotionally in completing the tastk

rapprochement (3rd phase of separation-individuation)

According to Margaret Mahler’s phases/sub-phases, Narcissistic personality disorders may be due to

inadequate soothing during the symbiotic phase (4 weeks-5 months)

inadequate refueling during separation-individuation (5-24 months)

Object constancy leads to the formation of

internalization

Allows child to have an internal representation of mother, allowing healthy separation exploration, and development of self-esteem

(internalizations which is formed by having having) object constancy

Unchanging, abnormal motor behaviors or action sequences without obvious reinforcement

Stereotypies

1st line treatment for stereotypies

Behavioral

What is the correct sequence of Bowlby’s stages of behavior when a young child is separated from his or her mother for an extended period of time

Protest (crying out)
Despair(giving up)
Detachment (emotional disengagement, indifferent about caregiver’s return)

3 features of intellectual disability

1. deficits in intellectual fxn
2. deficits in adaptive fxning
3. onset before age 18

In order to qualify for deficits in adaptive functioning, they have to result in

failure to meet developmental and sociocultural standards for personal independence and social responsibility
limit finding in 1 or more ADL

Valproic Acid has FDA indications for the treatment of ______ in the pediatric population

mania (12 years old and up)
seizure disorder

Therapeutic divalproex sodium levels in acute mania (adults)

50-120mcg/ml

Absolute contraindications for valproic acid use

hepatic disease/severe hepatic dysfunction
known urea cycle d/o
pregnancy
PCOS

Common side effects of divalproex sodium

alopecia
GI
neurological
opthalmological
rash

Divalproex sodium is _______ protein bound

80-90%

Maximum recommended dose of Divalproex sodium is

60mg/kg/day

What neurotransmitter is stored in large dense-core vesicles, modulate neuronal communication by acting on cell surface receptors, and are not recycled into cell after secretion

Somatostatin
Hypothalmic-releasing hormones
Endorphins
Enkephalins
Opioids

Small molecule transmitters that are stored in clear, small membrane-ground granules called synaptic vesicles and mediate fast synaptic transmission (and are recycled)

GABA
Glutamate
Glycine
Acetylcholine
Serotonin
Dopamine
NE
Epinephrine
Histamine

Which neurotransmitters are involved in the promotion of sleep?

Adenosine
GABA

Which neurotransmitters promote wakefulness

Dopamine
Histamine
Acetylcholine
Glutamate

Produced by cells in the hypothalamus and basal forebrain promoting NREM sleep

Adenosine

Neurotransmitters that promotes wakefulness

Dopamine
Acetylcholine
Norepinephrine

Setting fires, vandalism, cruelty to animals, and criminal behavior are characteristic of

Conduct disorder

Oppositional defiant disorder is more prevalent in which gender prior to adolescence?

1.4x more in males

Oppositional defiant disorder is more prevalent in which gender during adolescence?

same prevalence

Adolescent moral decision making is impacted by

limited impulse control
self-absorption
vulnerability to peer pressure

Dose of SSRI in OCD may be up to ______ the normal dose in depression

4X

IV immunoglobulin and penicillin are the treatment

PANDAS (Pediatric Autoimmune Neuropsychiatric Disorders Associated with Streptococcal infections)

Use of beta-agonists (used in tx of asthma), lead poisoning, anxiety, depression are part of the differential diagnosis for

ADHD

A child should be able to throw a ball by

18 months

A child should be able to scribble by

18 months

At 16 weeks a baby should be aware of

a strange situation

Average age of milestone of sitting up

4 months

At what age should a baby understand peek-a-boo and other simple games

40 weeks

Most common comorbidities of Fragile X are

ADHD (most males, 30% females)
social anxiety disorder (75% males)
autism (30%)

The IQ is typically 35-70 less than average and decreases with increased age

Fragile X

________ of females with Fragile X genes have decrease intelligence

33%

The brainstem is comprised of the

pons (metencephalon)
medulla ablongata (myelencephalon)
midbrain (mesencephalon)

The forebrain is composed of the

thalamus
hypothalamus
cerebrum

The midbrain is composed of the

corpora quadrigemina
aqueduct of Silvius

The hindbrain is composed of

medulla oblongata (brainstem)
pons (brainstem)
cerebellum (behind the brainstem)

Speech disorders, vestibular disturbances, dysphagia, respiratory disturbance, and abnormal consciousness are a few examples of ______ disorders

brain stem

Complete loss of brainstem function is called

brain death

Centre of temperature regulations

hypothalamus

Eating of feces

Coprophagia

_____ therapy is preferred in pica

Behavioral (limiting access to means)

Risk factors for conduct disorder

large family size
parental criminality
neglect
physical or sexual abuse
family hx SUD

Prevalence of conduct disorder

2-10%, mean 4%

One of the earliest and most specific symptom of conduct disorder

Childhood animal cruelty

Transitional objects are

physical representation of their mother at a time when they have not yet internalized their mother (stuffed animal, blanket)

Children usually surrender their transitional object by age

5

Major depression increases in prevalence with

age

Most prevalent child psychiatric disorder

ADHD (6.8%)

____% of adolescents demonstrate illicit drug use disorder in past year

4.7

____% of adolescents demonstrate alcohol use disorder in past year

4.3

____% of adolescents demonstrate cigarette dependence past month

2.8

____ % of children have autism spectrum disorder

1.1

_____ % of children with Tourette syndrome (age 6-17)

0.2

% of 3-17 year olds with anxiety

3.5

% of 3-17 year olds with depression

2.1

Approximately how many genes make up the human genome?

25,000

extensive drug extraction and metabolism in the liver immediately after absorption from the gut

First pass elimination

Fraction of the drug that reaches the systemic circulation and is available to exert a biological effect on target tissues

Bioavailability

Maximum plasma concentration of the drug

Cmax

Healthy balance between flexibility and stability

adaptability

Balance between closeness and separateness

Connectedness

Rumination is associated with both

gastrointestinal diseases
behavioral problems

Most individuals with DSM dx of Intellectual disability, profound, have

identified neurological condition(s)
considerable sensorimotor impairment

If no restricted or repetitive behavior patters (RBBs) are present but other criteria for Autism spectrum disorder are met, what is the diagnosis

Social communication disorder

A stage in the lifespan of an organism when it acquires developmental skill that is indispensable to life or survival

Critical period

General time during which an individual is more receptive to some types of environmental stimuli, usually because the nervous system development is especially sensitive to those stimuli

Sensitive period

“Learning is a cognitive process that takes place in a social context and can occur purely through observation or direct instruction, even in the absence of motor reproduction or direct reinforcement.”

Social learning theory

Detects diseases early when pts are asymptomatic and address tx to halt its progression (ie screening tests)

Secondary Prevention

Keeps a disease from occurring by removing its causes (counseling about lifestyle changes, suicide prevention programs, immunizations)

primary prevention

Main components of traditional behavioral couples therapy

Behavioral exchange
communication training
problem-solving

Cognitive restructuring and imaginal exposure are components of _______-

cognitive behavioral therapy

Role reversal is used in some forms of _________

family therapy

4 specific problems areas in individual interpersonal psychotherapy

grief or complicated bereavement
Role transition
Role dispute
Interpersonal deficits

when a pt expresses suppressed feelings, ideas or events to other group members

Ventilation

process of imparting a sense of optimism to group members and the ability to recognize that one has the capacity to overcome problems

inspiration

In integrative behavioral couples therapy, the technique of unified detachment relates to which of the following concepts

mindfulness

technique in integrative behavioral couples therapy that involves evocation of strong emotions in the dyad

empathetic joining

A result of empathetic joining, unified detachment, and tolerance building

emotional acceptance

Another technique used in integrative couples therapy that involves helping dyad members to be more understanding of upsetting behaviors

Tolerance

Believed to be the most important factor related to positive group therapeutic effect

cohesion

Conscious awareness and understanding of one’s own psychodynamics and symptoms of maladaptive behavior. Can be either intellectual or emotional

Insight

Statement made to promote an insight, which is the conscious realization of unconscious conflict

interpretation

Offers a connection between statements or behaviors but does not explain motivation

Observation

Increased risk for sporadic (non-familial) schizophrenia is associated with mutation rate in

Advanced paternal age (2x rate compared to advanced maternal age)

Mastery of conservation occurs at what Piaget childhood stage?

concrete operational stage (age 7-11)

Understanding that redistributing an object does not affect its mass, number or volume)

Conservation

Decentering occurs at what Piaget childhood stage

Concrete operational (age 7-11)

Taking into account multiple aspects of a problem to solve it

Decentering

Understanding that numbers or objects can be changed and returned to their original state

Reversibility

Reversibility occurs at what Piaget childhood stage

Concrete operational (age 7-11)

The ability to sort objects mentally and to recognize the relationships among various things in a series

Transitivity

Transitivity occurs at what Piaget childhood stage

Concrete operational (age 7-11)

The rate of psychopathology among those with an intellectual disability is roughly ________ than in the general population

4x higher

To dx Tourette’s, symptoms must be present before what age

18

Conduct symptom review
Relate symptom onset to dispute
take hx of relationship
dissect role expectations
focus on correction of nonreciprocal expectations

IPT sequence for interpersonal disputes that cause depression

Review depressive sx
Relate them to the death of significant other
reconstruct lost relationship
explore negative and positive feelings
consider options for increasing social support

IPT sequence for depression related to grief

Review symptoms and relate them to social isolation or unfulfillment
Review past relationships
Explore repetitive patterns
Discuss positive and negative feelings about therapist and explore maladaptive patterns.

IPT sequence for interpersonal deficits that lead to depression

Review symptoms
Relate symptoms to life change
Review positive and negative aspects of new and old roles
Review losses
Ventilate feelings
Find new role options

IPT sequence for patients with role transitions

Which type of group therapy indicated for neurotic disorders focuses on present and past life situations, as well as intragroup relationships

Psychodynamic group

Conversion of unconscious wishes into their opposites. The behavior is opposite of what one really desires or feels

Reaction formation

Which of the following IPT focus areas most accurately describes conflicts in significant relationships?

Role dispute

The Maudsley model is used to treat

Anorexia nervosa (family therapy for pts up to 19 years of age)

In this type of family therapy, the family plays an active ole in tx of anorexia pt with goal of weight restoration and the eventual return to control regarding eating habits of the patient

Maudsley Model

According to Minuchin, what factor accounts for family dysfunction symptoms?

symptoms arising from structural imbalances in family system

Family of origin issues are addressed in theses psychotherapeutic modalities

psychodynamic
transgenerational family therapy
Bowen’s family therapy

This type of family therapy emphasizes the role of biologically based disorders

Psychoeducational family therapy

A shared projection process is part of these psychotherapeutic modalities

Psychodynamic
Transgenerational family therapy

Pt putting on contact lenses for first time blinks eyes before the contact lens touches her eyes. What is the pathway that explains this reflex arc.

CN V afferent CN VII efferent

Early onset bipolar disorder is less responsive to

Lithium

Steven Johnson syndrome occurs in ____ of children on lamotrigine

1%

People avoid pain discomfort and strive for gratification and pleasure

Pleasure Principle

Process that occurs when repressed memory and associated affect is brought into consciousness

Abreaction

Unconscious intrapsychic process that regulates basic biological drives and maintains psychological homeostasis

Defense mechanism

Unconscious conflict between opposing wishes or wishes and prohibitions

Defense mechanism

Applied behavioral analysis (ABA) effectively treats

self-injurious behavior (especially with autism)

Which school of family therapy has a here-and-now focus and encourages change through growth experiences

Experiential

According to Kohut, diagnosis of narcissistic personality disorder is based on

predominant transference estabilished (ie. self-object transference involving mirroring and idealization)

According to Kernberb, the diagnosis of narcissistic personality disorder is based on

defenses the pt uses
(splitting, projective identification, primitive idealization)

Sx of resistance in psychoanalysis

silence
asking unrelated questions
intellectualizing things
being late for appointments
critically analyzing the reasons for treatment methods
censoring thoughts and ideas

Coming up with an incorrect conclusion based on a previous experience

Arbitrary inference

Taking a small detail out of context to make an entire experience negative

Selective abstraction

In family therapy the term triangulation is tied to the work of

Murray Bowden

Coming to a generalized conclusion based on a single event

Overgeneralization

Several 3.5 year old children are playing in a sandbox with toys. What kind of play behavior are they most likely to exhibit. They share the same toys but still play separately.

Associative play

2 year old children playing.

Parallel play (no interaction with each other)

4 year old children playing

Cooperative play (start to interact with each other)

Preschool children (age 2-5) primarily use

logical thinking

Egocentrism and magical thinking are prominent in

preschool children

Preschool children have difficulty with ____ and can believe if 2 events occur, 1 caused the other

causation

Categorizing oneself or one’s personal experiences into rigid dichotomies

absolute thinking

Who believes that narcissist and borderline patients share core features of borderline personality organization which consists of pathological internalized object relation

Kernberg

In treating people with narcissistic personality, _____ tended to be more supportive and introspective, while ______ gently confronted defenses more actively

Kohut
Kernberg

What is the primary, direct cause of neuronal loss in an ischemic stroke

Excessive increases in excitatory neurotransmitters (excitotoxicity)

Development of imagination and symbolic thinking, including language happen during the

preoperational stage

Egocentrism is characteristic of the __________ of childhood development

preoperational stage

Use of logic and interpreting experiences objectively are characteristic of the _______ of development

concrete operational

During the _______ of development, a child is unable to consider 2 dimensions

preoperational

What happens to Fluoxetine’s peak plasma concentration when taken with food

delayed but not decreased/increased

What happens to Paroxetine’s peak plasma concentration when taken with food

increased by 6%

What happens to Sertaline’s peak plasma concentration when taken with food

increased by 25%

What happens to the peak plasma concentration of Citalopram when taken with food

no affected

Provide scaffolding of brain

astrocytes

form blood-brain barrier

astrocytes

guide neuronal migration during development

astrocytes

remove neurotransmitters from within the synaptic cleft (preventing toxic buildup)

astrocytes

regulate the external chemical environment

astrocytes

capable of cell-cell signaling

astrocytes

produce myelin sheath that speeds conduction of action potentials along axons in the CNS

Oligodendrocytes

Produce myelin in the peripheral nervous system

Schwann cells

Macrophages of the brain

microglia

make up the walls of the ventricles

Ependymal cells

Secrete CSF

ependymal cells

help to circulate CSF

ependymal cells

types of glial cells

astrocytes
oligodendrocytes
microglia
ependymal cells

Medical conditions that cause ASD can be identified in ______% of cases

<5

Medical conditions that are best established as probable causes of ASD include

Rett syndrome
fragile x syndrome
tuberous sclerosis
abnormalities of chromosome 15 (involving 15q11-13 region)

Mitochondria dysfunction can be identified in ______% of autism cases

up to 5 (possibly more, 500x higher than general population)

The ________ gene associated with fragile X syndrome is associated with ASD

FMR1

Pt in car accident hit head on dashboard and suffered cribriform plate fx. He has clear fluid dripping from his nose. What else might you expect in this pt

loss of olfaction

Risk factors for child/adolescent suicide

affective d/o
substance abuse
living alone
prior suicide attempts
male gender
gay/lesbian/bisexual
age>16

word articulation typically lags behind ______

vocabulary

Most articulation errors end by age

4

Children should start crawling by ________ and walking by _______

8 months
12 months

By age 3, regarding speech, children should be______

using real sentences with grammatical function

By age 3, a child’s vocabulary should consist of around _______ words

895

When treating conversion disorder, doing this is contraindicated

confrontation about the symptom

How long do conversion disorders last for

usually remit spontaneously within 2 weeks and it is unusual that they persist for a year.

What is seen primarily with the initiation of atypical neuroleptics

sedation

What is seen after months of atypical neuroleptic therapy

Tardive dyskineasia

3 risk factors for child abuse

Prematurity (especially with incessant crying)
developmental disability
physical disability

What gender has a higher risk of child abuse

Male infants

At what Piaget stage do children understand conservation

operational stage @ age 7

In what way does necrosis differ from apoptosis

inflammatory response

Unlike necrosis, the process of apoptosis does not stimulate an

inflammatory response

Which neurobiological abnormality is associated with autism?

Decreased formation of Purkinje cells in cerebellum

The ASD symptoms in Rette are

Severe, profound impairment

Its with ASD have _______ overall brain size

increased

Patients with ASD experience ______ growth of head during infancy

accelerated

What is correct regarding characteristic dysmorphic features of Williams syndrome

elfin facies
starburst irises

Common comorbidities of Williams syndrome

anxiety disorders
depression

Medical issues of Williams syndrome

supravalvular aortic stenosis
renal artery stenosis
HTN

What is the genetic anomaly with Williams syndrome

genetic deletion occurs at chromosome 7

Autistic child cannot tell you what objects are when she holds them with her eyes closed. This may be due to

astereognosis (may be d/t lesion in parietal lobe)

SIGECAPS stands for

Sleep disorder
Interest deficit (anhedonia)
Guilt (worthlessness, hopeless, or regret)
Energy deficit
Concentration deficit
Appetite: increased or diminished
Psychomotor retardation or agitation
Suicidality

In subsequent suicide attempts, >80% suicide attempters

use 2 or more methods

Maintenance therapy of an antidepressant is recommended for patients with additional risk factors for recurrence such as

presence of residual symptoms
ongoing psychosocial stressors
early age at onset
family history of mood disorders

How long must sx be present in children and adolescents to be diagnosed as cyclothymic disorder

1 year

According to the APA on MDD, what is the correct definition of remission during acute-phase tx from an episode of major depression

At least 3 weeks of absence of both sad mood and reduced interest and no more than 3 remaining symptoms of major depressive episode.

In the outpatient setting about _______% of children referred for depression are suicidal

50

FDA approved for treatment of bipolar depression in pt on lithium

Quetiapine
Olanzapine plus fluoxetine (in combo)
Lurasidone

Which medication has been shown to reduce relapse rates significantly and to diminish the severity and duration of bipolar disorder in postpartum women when used as a prophylaxis (at 36 weeks of gestation or no later than the first 48 hours following delivery)

Lithium

When should you start prophylactic lithium to reduce relapse rates significantly and diminish the severity and duration of puerperal illness?

36 weeks gestation or no later than the first 48 hours following delivery

A patient is on carbamazepine and amitriptyline. He c/o of dry mouth and urinary retention. His antidepressant dose has not change? What is the most likely explanation of the carbamazepine dose?

It was decreased

A patient is on phenytoin and amitriptyline. He c/o of dry mouth and urinary retention. His antidepressant dose has not change? What is the most likely explanation of the phenytoin dose?

It was decreased

amitriptyline. He c/o of dry mouth and urinary retention. His antidepressant dose has not change? What is the most likely explanation of the oxcarbamazepine dose?

It was decreased

Barbituates and vigabrain have an increased risk of depression, suicidal ideation and suicidal behavior in

epileptic patients (FDA warning level)

Anticonvulsants like ______ are useful in treating mood disorders

carbamazepine
valproate
lamotrigine

Clozapine can reduce suicidality in patients with

schizophrenia
schizoaffective
(not bipolar)

Regarding cultural aspects, minorities are less likely to receive tx for MDD, specifically _____ and _____

ECT
psychotherapy

According to the DSM-5, how many overlapping sx must be present to include the “mixed-features” specifier in the dx of depression in a manic/hypomanic episode.

3

What is the completed suicide rate in patients with bipolar I

10-15%

People with a first degree relative with MDD are ______ x more likely to have MDD

1.5-3X

People with a family hx of depression generally have

earlier age of onset
more chronic course
recurrences

When unipolar pts are tx with antidepressant, the rate of experiencing a “switch” into mania is estimated to be _____ % per year

3-4

According to the Texas STAR-D algorithm, an agent with partial response can be ______

augmented or changed

If new onset Bipolar affective disorder (BAD) starts around ages 50-59, it is usually d/t

medical condition
illicit drugs
medication

These antidepressants have the lowest level of exposure in breast milk.

sertraline
paroxetine
nortriptyline
imipramine

19 year old with first manic episode, started on lithium. On day 2, experiences motor excitement, mutism, and stereotypic movement. Failed lorazepam trial. What treatment is indicated next to tx his sx.

ECT

Up to ______ % of manic pts will develop catatonic features indicated by negativism, withdrawal, starting, immobility/stupor, mutism, posturing, grimacing, stereotypes, mannerism of non-purposeful excitement, undirected combativeness, unexplained impulsive behavior, echopraxia, echolalia, motor excitement, bizarre grimacing, and wavy flexibility

33%

When the diagnosis of catatonia has been made, the use of ______ should be generally avoided

antidepressant and/or antipsychotic medications

________ can worsen catatonia d/t parkinsonian side effects and can increase risk of ______

Antipsychotics
neuroleptic malignant syndrome

Patients with bipolar disorder may account for 25% of which of the following?

Completed suicides

Patients with bipolar are _____x more likely to commit suicide than the general population

15

Suicidality is commonly seen in bipolar pts when they are in a _____ or _____ condition

depressed
mixed

Pts taking selegiline may test urine positive for which of the following drug

methamphetamine

Selegiline is metabolized by the body into

I-methamphetamine and I-amphetamine

Selegiline is a substituted phenethylamine used for the treatment of

early-stage Parkinson’s disease, depression, and dementia.

In normal doses selegiline is a _______ and in larger doses it is a _________

selective irreversible MAO-B inhibitor

MAO-A and MAO-B inhibitor

What percentage of patients with Parkinson’s disease have some degree of depression

40-50%

What sleep abnormality is present in MDD

Decreased slow-wave sleep
Sleep onset latency increased
Total sleep time decreased
REM latency shorter with number of eye movements increased

What substance is most commonly associated with those who die by suicide?

Alcohol

Rapid-cycling requires at least _____ major depressive, manic, or hypomanic episodes that met criteria in the past year. These episodes must be distinct (with at least a partial remission for at least______ months or switch to an opposite type of episode)

4
2

Lithium use in first trimester can lead to

Ebstein’s anomaly and other cardiac malformations,

Valproate sodium is not safe in pregnant women because it can cause

intrauterine growth retardation
neural tube defects

SSRIs cause more ______ in the elderly

SIADH (3x risk)

When using TCAs in elderly, ____ and ______ are preferred

desipramine
nortriptyline

Antidepressant for elderly who has insomnia and weight loss

mirtazapine

Blood levels of medications like TCAs are _____ in elderly patients

higher

Paroxetine has FDA indications for what in children and adolescents?

Nothing

Paroxetine is _____% plasma bound and ______% excreted in urine

95
66

The half life of paroxetine is _______, increasing risk of severe discontinuation syndrome

21 hours

Antidepressants that can increase insomnia and activation

Fluoxetine
bupropion

Antidepressant that can cause activation but is less likely to cause activation compared to Fluoxetine and Buproprion

Sertaline

What is the minimum duration of symptoms for diagnosing a hypomanic episode?

4 days

In MDD, seasonal pattern, some ____ occurs after the depressive episode

hypomania

When taking an MAOI, need to avoid these foods/drinks

cheese
caviar
liver
smoke/picked/cured fish and meat
ripe avocados
canned figs
yeast extract
Chianti red wine
fava beans (not lima)
raisins
prunes
bear with yeast

When taking MAOI, need to keeps these foods in moderation

chocolate
coffee

Medications to avoid when taking MAOI

cold meds
allergy meds
decongestants
cough meds
stimulants
meperidine
SSRI
bupropion
mirtazapine
nefazodone
trazodone
venlafaxine

Who has the highest suicide rate of any age and ethnic group

Caucasian men > 75 yrs old

What is the mean age of onset for the first manic, hypomanic, or major depressive episode of bipolar I disorder

18 years

What percentage of patients with bipolar I disorder have >1 manic episode?

>90

Inpatient methods for reducing suicide

Non-weight bearing fixtures and curtain rods
Cordless phone
Jump-proof windows
Short cords on adjustable beds
Confiscate belts/shoelaces
Barricade-proof doors

Which form of family therapy externalizes the main problems and identifies exceptions to the dominant story

Narrative

Externalizes problems and focuses on the effects on people’s lives rather the on problems as inside and part of people.

Narrative therapy

A functional analysis is completed in

behavioral therapy

Skills such as communication, problem-solving, and coping strategies are explored

psychoeducation

Circular questions are used in _____

systemic therapy

cognitive error where a part of a situation is picked out and exaggerated

maximization

Enmeshment involves a failure to achieve appropriate

interpersonal boundaries

Who developed the model of therapy resolving around triangle of conflict, defense work, and forming a portrait to resolve dynamic pathologic defenses?

Malan

In _______’ anxiety provoking therapy, the therapist acts as teacher

Sifneos

_______ developed intensive short term dynamic psychotherapy

Davanloo

In ______’s existential therapy, the therapist acts as an empathetic helper

Mann

In the launching stage, the main task for the couple is to

reevaluate their marriage and career issues as their parenting roles diminish

An individual demonstrating good coping skills is

quite composed and vigilant in avoiding emotional extremes when they are inappropriate, in an effort to avoid impaired judgment

The term “goodness of fit” as described by Chess and Thomas

Match or mismatch between a person’s temperament and features of his or her environment. Expectations and demands are in accord with the organism’s own capacities, characteristics and style of behaving. Socioeconomic levels, specific values and cultural differences must be taken into consideration

In DBT, dialectics refer to which of the following?

philosophy and approach to communicating with patients
(philosophical view of the nature of reality emphasizing the non-linear and dualistic nature of the world)

A 4 year old runs to mother after riding bike without training wheels and says “Look mommy, I did it all by myself.” Which basic conflict best described this child?

Initiative vs guilt

Relational theory focuses on the real relationship between pt and therapist to understand and relieve conflict and social inhibition and to achieve social intimacy. Which theorist is associated with this type of therapy?

Jean Baker Miller

Coming to an incorrect conclusion based on a previous experience

Arbitrary inference

In custody cases ___________ is necessary to provide an unbiased, objective eval of the situation

a forensics evaluation by a forensic evaluator

In couples therapy, the therapist looks for ways to understand how the marriage is “stuck”. Which approach is the therapist using?

Structure-strategic Approach

The therapist tries to help the patient get out of persistent narrow role by encouraging connective emotional experiences in the here-and now. Which approach is the therapist using?

Experimental humanistic approach

The therapist encourages cognitive restructuring to relieve marital tension. Which approach is the therapist using?

Behavioral approach

The therapist focuses on the relationship between one’s role in the family of origin and the current couple difficulty. Which approach is the therapist using?

Transgenerational approach

Negative interpersonal experiences can lead to

depression in vulnerable individuals

Emphasizes the role of underlying psychosocial conflicts, not psychosexual

Psychodynamic therapy

In psychodynamic therapy, “working through” focuses on which of the following

Identifying patterns of defense mechanisms and object relations. This is restricted to patient and therapy

Identification of dysfunctional cognitions is part of

cognitive-behavioral therapy

What is learned in concrete operational stage

conservation
reversibility

Realization that one thing can be turned into another and back again

Reversibility – learned in concrete operations stage

Abstract thinking is obtained during this Piaget stage

Formal operational

The eclectic brief therapy of Budman and Gurman rests on interpersonal development and existential focus (IDE). What is included in his method

losses
developmental dyssynchonies
interpersonal conflicts
symptomatic presentations
personality disorders

A characteristic of their therapy is the belief that maximal benefit occurs early and that is the opportune time for change

IDE by Budman & Gurman

Traditional behavioral couples therapy does not

emphasize cognition

In Behavioral couples therapy based on operant condition -____

decreases in social punishers decreases dyadic stress

To understand the relationship between interpersonal experiences and depression, what would a therapist do with her patient?

Construct a timeline

Conducting an interview focusing on early childhood experiences is consistent with

psychodynamic psychotherapy

Chain analysis is a technique employed in

DBT

Thought-mood logs are useful in

cognitive behavioral therapy

Involves construction of an illness (depression) timeline reflecting major mood changes and significant interpersonal events

Individual interpersonal psychotherapy

What is it called during individual interpersonal psychotherapy when the therapist helps the patient accept the medical diagnosis of depression and its seriousness

Inducing patient into sick role

Codependent behavior where one party supports the maladaptive behavior of a second party

Enabling

Exposing patients to anxiety provoking or feared situations all at once until anxiety and fears subside

Flooding

Behavioral therapy has been shown to ameliorate signs related to

eating disorders
mood disorders
phobias
OCD/OCPD

Gradual exposure to anxiety-provoking situations, introduced by “imagined exposure” is helpful for

agoraphobia

Common form of relaxation training

Progressive muscle relaxing

Progressive muscle relaxing is helpful for pts with

anxiety disorder
pain disorders

Which type of therapy has been effective for women with opioid dependency?

DBT

Inclusion for ________:
real desire for relief
moderate emotional distress
specific/circumscribed problem
ability to commit to tx
hx of positive relationship and function in one area of life

brief therapy

Exclusion criteria for __________:
acute risk self harm
active psychosis
acute/severe substance abuse

brief therapy

Erikson’s task of basic trust

Trust vs Mistrust
(Birth-18 months)

Erikson’s task of control and independence

Autonomy vs Shame
(18 months-3 years)

Erikson’s task of taking control of the environment, purpose

Initiative vs guilt
(3-6 years)

Erikson’s task of confidence, competence, social skills

Industry vs inferiority
(6-12 years)

Erikson’s tasks of formation of identity, devotion

Identity vs role confusion
(12-18 years)

Erikson’s tasks of forming a relationship, commitment

Intimacy vs isolation
(18-35 years)

Erikson’s task of building a family, having a productive career

Generatively vs stagnation
(35-65 years)

Erikson’s tasks of viewing life as meaningful and fulfilling

Ego integrity vs despair
(65 years to death)

A therapist should limit silences in

brief therapy

Lazarus’s conceptualization of coping

Self-exploration
Self-instruction
Self-correction
Self-rehearsal

What type of psychotherapy has the least empirical data to supports its use in schizophrenia

psychoanalysis

Leave a Comment

Scroll to Top